#5 Rosh Review

Réussis tes devoirs et examens dès maintenant avec Quizwiz!

Question: In CSF analysis, what does xanthochromia indicate?

Answer: Red blood cells in the subarachnoid space.

Question: What are three common forms of diabetic neuropathy?

Answer: Autonomic (gastroparesis, orthostasis, constipation), acute mononeuropathy (CN palsy, foot drop) and distal, symmetric, sensory > motor peripheral neuropathy. Rapid Review Diabetic Ketoacidosis Caused by precipitating factor in known diabetic Hyperglycemia + ketonemia + acidosis Anion gap metabolic acidosis with respiratory compensation Corrected Na= add 1.6 mEq/L for each 100 mg/dL in serum glucose IVF, insulin infusion K+: monitor and replace as needed Mortality in children: cerebral edema Mortality in adults: sepsis, cardiopulmonary complications

Question: What injury can occur if Hemoccult drops are instilled in the eye?

Answer: Severe alkali burns can occur. Rapid Review Subconjunctival Hemorrhage Coughing, sneezing, minor trauma Under conjunctiva and stops at limbus Reassurance

Question: If vesicles are noted in the external ear canal of a patient with vertigo, what is the appropriate therapy?

Answer: Acyclovir for herpes zoster oticus (Ramsay Hunt Syndrome). Rapid Review Labyrinthitis Patient with a history of recent upper respiratory infection Complaining of sudden and persistent onset of vertigo and hearing loss lasting several days to a week

Question: What medication is used to prevent attacks of recurrent gouty arthritis?

Answer: Allopurinol. Rapid Review Gout Patient will be a middle-aged man Complaining of acute onset of pain in the first MTP (Podagra) Labs will show needle-shaped crystal with negative birefringence Most commonly caused by uric acid crystals Treatment is: Acute: NSAID's Chronic: allopurinol or colchicine Comments: can be triggered by loop and thiazide diuretics

Question: For non-crusted scabies, how should permethrin cream be applied?

Answer: Apply all over body overnight once and reapply in 14 days if mites are still present. Rapid Review Scabies Patient will be complaining of severe pruritus that is worse at night PE will show small papules, vesicles and burrows in the webbed spaces of the fingers and toes Diagnosis is made by microscopic visualization Most commonly caused by Sarcoptes scabiei hominis Treatment is permethrin 5%

Question: What is malignant hypertension?

An outdated term which has been replaced by hypertensive emergency. Rapid Review Hypertensive Emergency Hypertension with acute end-organ system injury Encephalopathy, cardiac ischemia, renal ischemia Objectives: reduce MAP 25% in first hour, normalize BP over the next 8 to 24 hours Reduction of MAP > 25% may cause end-organ ischemia IV antihypertensives (labetalol or nicardipine)

Question: Operative hemorrhoidectomy results in what rate of recurrence?

Answer: 2-5%. Rapid Review Hemorrhoids Internal Proximal to dentate line Painless bleeding External Below dentate line Visible Painful Initial rx: WASH (Water (sitz bath), Analgesics, Stool softeners, High fiber diet) Thrombosed hemorrhoid rx: excision with elliptical incision

Question: In addition to isoniazid, what other drugs can lead to niacin deficiency?

Answer: 5-fluorouracil, pyrazinamide, 6-mercaptopurine, hydantoin, ethionamide, phenobarbital, azathioprine, and chloramphenicol.

Question: How long do patients with untreated polycythemia vera generally survive?

Answer: 6-18 months from time of diagnosis. Rapid Review Polycythemia Vera Patient will be complaining of headache, dizziness, pruritus after showering PE will show hypertension, splenomegaly Labs will show increased RBC mass, overproduction of all cell lines, increased Hgb Most commonly caused by mutation of the Janus kinase 2 gene (JAK2) Treatment is phlebotomy, hydroxyurea, aspirin

Question: At what age should breast cancer screening for women without risk factors stop?

Answer: 74 years.

Question: What is the treatment for erysipelas for patients whom are allergic to penicillin?

Answer: A first-generation cephalosporin or macrolide, such as erythromycin or azithromycin, may be used if the patient has an allergy

Question: What is an Austin-Flint murmur?

Answer: A variant of aortic regurgitation, the Austin-Flint murmur is best heard with the bell at the apex, is more low-pitched and rumbling, and can be presystolic or mid-diastolic. Rapid Review Aortic Regurgitation Acute MCC: endocarditis Chronic MCC: rheumatic heart disease Blowing diastolic murmur at left sternal border Pulse pressure: normal (acute), widened (chronic) de Musset sign: head bobbing with systole Quincke's pulse: prominent nail pulsations Duroziez murmur: "singsong" murmur over femoral artery Austin-Flint murmur: mid-diastolic murmur in severe AR Rx objective: ↓ afterload

Question: What syndrome is associated with a VSD that results in right-to-left shunting?

Answer: Eisenmenger syndrome. Progressive high pulmonary vascular pressure alters flow from left to right through the VSD to right to left, leading to cyanosis. Rapid Review Acyanotic Congenital Heart Disease ASD: fixed split S2 VSD: most common congenital heart defect, harsh holosystolic murmur at LLSB, complication: Eisenmenger syndrome PDA: machine-like murmur, Rx: indomethacin Coarctation of the aorta: upper extremity > lower extremity BP/pulse, CXR: rib notching, Rx: PGE1 Echocardiography

Question: What upper extremity fracture often compromises the radial nerve?

Answer: Humeral shaft fracture. Rapid Review Peripheral Neuropathies Sciatic: buttock injury, hip dislocation, ↓ knee flexion, foot drop, rx: ankle splint Common peroneal: proximal fibula injury, footdrop, rx: ankle splint Radial: crutches, wrist/finger drop, rx: wrist splint Ulnar: elbow injury, ↓ finger adduction/thumb grasp, 4th/5th digit paresthesias Lateral femoral cutaneous: inguinal ligament entrapment, upper thigh dysesthesia/numbness

Question: What is the most common cause of pericarditis?

Answer: Idiopathic. Rapid Review Acute Pericarditis Idiopathic > viral (Coxsackie) Pleuritic chest pain radiating to the back Pain ↓ with leaning forward Pericardial friction rub ECG: diffuse STE, PR depression NSAIDs

Question: What is the recommended treatment for dyshidrotic ecematous dermatitis?

Answer: Steroids can be administered topically, intralesionally or systemically depending on the severity. PUVA (psoralen and UVA light) may also be beneficial. Rapid Review Dyshidrotic Eczema Patient will be complaining of intense pruritus on their palms and sides of the fingers PE will show vesicles that appear to contain "grains of tapioca" Treatment is avoidance of long exposure of the hands to water, topical corticosteroids for acute flares

Question: What is the most commonly isolated pathogen from a peritonsillar abscess?

Answer: Streptococcus pyogenes.

Question: Infants of mothers with myasthenia gravis are at risk for which disease?

Answer: Transient neonatal myasthenia gravis. Rapid Review Myasthenia Gravis Patient will be complaining of proximal muscle weakness, ptosis, diplopia that is worse at the end of the day PE will show ice test improves sx Diagnosis is made by edrophonium (tensilon) test, EMG Most commonly caused by autoimmune destruction of acetylchoine receptors Treatment is acetylcholinesterase inhibitors, such as pyridostigmine Comments: associated with thymoma

Question: Which organism causes condyloma latum?

Answer: Treponema pallidum (syphilis).

Question: Which type of renal calculus is radio translucent?

Answer: Uric Acid stones. Rapid Review Nephrolithiasis Patient will be complaining of flank pain radiating to groin PE will show a patient that won't lay still and hematuria Diagnosis is made by helical CT Most commonly caused by calcium oxalate Struvite: staghorn calculi, urease producing bacteria Uric acid: radiolUcent on xray, gout Cystine: children with metabolic diseases Most common location is the ureterovesiclular junction (UVJ) Treatment is: < 5 mm: likely to pass spontaneously > 8 mm: unlikely to pass, lithotripsy

A 15-year-old boy presents to the emergency department with lower back pain. He reports heavy lifting three weeks ago, with gradual onset of pain and no relief with symptomatic care. The pain is localized to the paraspinal muscles over his lumbar back. He reports normal urine output. Baseline laboratory tests are notable for a sodium of 140, potassium of 5.2, chloride of 110, bicarbonate of 25, BUN of 20, and creatinine of 2.3. A urinalysis shows hyaline casts, 5 white blood cells, and 1+ protein. Which of the following is the most likely pathophysiology of his laboratory findings? Acute interstitial nephritis Acute tubular necrosis Hypovolemia Obstructive nephropathy

Correct Answer ( A ) Explanation: This patient has acute renal failure (ARF), exhibited by his elevated creatinine level. The common causes of ARF can be subdivided into three categories: prerenal, intrinsic renal, and postrenal. Intrinsic renal pathology can be further subdivided into glomerular disease, interstitial disease, and tubular disease. This patient has intrinsic renal disease, as his BUN to creatinine ratio is less than 15 and his urine output is normal. The most likely etiology of his intrinsic renal disease is acute interstitial nephritis (AIN) secondary to overuse of non-steroidal anti-inflammatory drugs for back pain. Many drugs and toxins can cause AIN. It typically presents one to two weeks following onset of the inciting agent, and may be associated with symptoms such as fever, rash, arthralgias, and eosinophilia. Urine output is preserved. Urinalysis may show white blood cells and hyaline or granular casts, as well as mild proteinuria. Removal of the offending agent often leads to recovery. Acute tubular necrosis (B) is another type of intrinsic renal damage. It is typically seen in critically ill children with sepsis, hypotension, or exposure to nephrotoxic medications. ATN is characterized by elevated creatinine and a urinalysis showing brown granular casts. Hypovolemia (C) causes prerenal ARF. This type of renal failure is exhibited by decreased effective circulating arterial volume, which leads to decreased renal perfusion and decreased GFR. The BUN to creatinine ratio should be elevated, and urinary casts are not commonly seen. Intrinsic renal damage may develop if hypoperfusion of the kidneys persists. Obstructive nephropathy (D) may be seen in children with posterior urethral valves, nephrolithiasis, and other urinary tract abnormalities. It is exhibited by decreased urine output and hematuria, and hydronephrosis is commonly seen.

A 27-year-old man with a history of asthma presents to your office for his annual exam. He reports that in the past month he has experienced wheezing and shortness of breath about once per week which resulted in using his rescue inhaler. He woke up once because of coughing. He has a peak flow meter and home readings have been 85-90% of his personal best. Which of the following best describes the classification of his asthma? Intermittent Mild persistent Moderate persistent Severe persistent

Correct Answer ( A ) Explanation: Asthma severity is determined by monitoring patient symptoms over the past 2-4 weeks, determination of the patient's current lung function and number of asthma exacerbations that require oral glucocorticoids each year. A step-wise approach is used to classify asthma severity and determine approach to treatment. Intermittent asthma is defined as having two or fewer daytime symptoms per week, two or fewer nighttime awakenings due to symptoms in the past month, use of short-term beta agonist rescue inhaler less than two times per week, no impairment in activities of daily living (ADLs) between exacerbations, home readings of lung function within the normal range (greater than or equal to 80% of normal), and zero or one exacerbation requiring oral glucocorticoids in the past year. Asthma is classified as mild persistent (B) with more than two daytime symptoms per week but less than daily, 3-4 nighttime awakenings in the past month, use of rescue inhaler more than twice in one week but less than daily, mild impairment in ADLs, home readings of lung function within normal range and two or more exacerbations in the past year requiring treatment with oral glucocorticoids. Moderate persistent (C) classification shows an increase in daytime symptoms to daily, nighttime awakenings more than once per week, daily use of the rescue inhaler, limitations in ADLs and home readings of lung function between 60-80% of normal. Severe persistent (D) is classified as asthma symptoms throughout the day, awakenings every night, use of rescue inhaler several times every day, severe impairment in ADLs and lung function less than 60% of normal.

A ten-year-old girl presents with drooping eyelids for the past month. The patient reports some difficulty completing her schoolwork due to blurry vision and hand weakness. On initial observation, her exam is notable for bilateral ptosis. Which of the following is the most likely pathophysiology behind her symptoms? Acetylcholine receptor antibodies Acetylcholinesterase deficiency Anti-thyroid antibodies Toxin exposure

Correct Answer ( A ) Explanation: Antibodies to the acetylcholine receptor are the most common cause of myasthenia gravis. Myasthenia gravis is a chronic, autoimmune disease characterized by fatigue of striated muscle. The first clinical signs are ptosis or extraocular muscle weakness or both. Later symptoms include bulbar muscle weakness, limb-girdle weakness, and hand weakness. This disease is notable for the rapid fatigue of muscles, which may be demonstrated by sustaining an upward gaze for 30-90 seconds or repetitive opening and closing of the fists. Symptoms increase throughout the day. Myasthenia gravis is a progressive disease and may be fatal due to respiratory muscle compromise or aspiration. A clinical diagnosis may be made with the administration of edrophonium chloride (Tensilon test), which causes immediate improvement in the ptosis and ophthalmoplegia. In infants, Neostigmine should be used to make the diagnosis, as edrophonium has increased risk of cardiac dysrhythmias in infants. Cholinesterase inhibitors are the first line of therapy. Plasmapheresis and IVIG, as well as thymectomy, can be used in refractory cases. Anticholinesterase deficiency (B) at the motor endplate is a less common cause of myasthenia gravis; this deficiency is associated with familial myasthenia gravis. Myasthenia gravis may be associated with Hashimoto's thyroiditis (C), and thyroid levels should be check in all patients. Treatment of hypothyroidism usually leads to resolution of myasthenia symptoms. However, this patient does not report concurrent symptoms of hypothyroidism. Toxin exposure (D) must be considered in the differential of neuromuscular blockade. These toxins include organophosphate chemicals, botulism, and tick paralysis. This patient has no history of exposure to these toxins.

A 25-year-old sexually active woman presents to the Emergency Department with a complaint of painful vulvar ulcers and a swollen inguinal lymph node. She denies dysuria and vaginal discharge. Which of the following is most consistent with the description of the offending infectious organism? Gram negative bacillus Gram negative diplococcus Obligate intracellular gram negative organism Spirochete

Correct Answer ( A ) Explanation: Chancroid is a common sexually transmitted disease in developing nations, but is relatively rare in the United States. Chancroid is caused by Haemophilus ducreyi, a gram negative bacillus, which has an incubation period of 4 to 10 days. Initially, a small pustule will form at the site of inoculation and progresses to multiple, painful ulcerations with sharply demarcated purulent bases. The number of ulcers and the pain associated with the ulcers are the distinguishing features of chancroid and differentiate it from syphilis. Chancroid is also associated with painful inguinal lymphadenopathy, termed a bubo, which is unilateral, large, painful, fluctuant and may become suppurative. Patients will frequently complain of dysuria, but this is typically due to urine passing over the exposed ulcers and not due to urethral inflammation. Diagnosis is made with culture and both herpes and syphilis should be ruled out. Treatment is with Azithromycin 1 gm orally once, ceftriaxone 250 mg intramuscularly once, ciprofloxacin 500 mg orally twice a day for three days, or erythromycin 500 mg orally four times a day for 7 days. Neisseria gonorrhoeae is a gram negative diplococcus (B) and the cause of gonorrhea, the second most commonly reported sexually transmitted disease. Gonorrhea causes urethritis, purulent discharge, epididymitis and orchitis, proctitis, and pelvic inflammatory disease. It may also disseminate and cause systemic symptoms, endocarditis, septic arthritis, tenosynovitis, and an acral pustular or petechial rash. It is not known to cause painful genital ulcers. Chlamydia trachomatis is an obligate intracellular gram negative organism (C), the most commonly reported sexually transmitted disease in the United States, and the infectious cause of chlamydial infections. Chlamydia can cause Lymphogranuloma venereum, which manifest as painless genital ulcers and tender inguinal lymphadenopathy. Syphilis, caused by the spirochete (D) Treponema pallidum, first presents as a painless chancre or genital lesion after a 3 week incubation period, and is easily confused with the painful lesion of chancroid. The chancre spontaneously heals in 3-6 weeks before the infection progresses on to secondary (characterized by a rash and lymphadenopathy) and tertiary syphilis which affects the nervous and cardiovascular system through neuropathy and widespread granulomatous lesions.

A 30-year-old man with ankylosing spondylitis presents for a routine physical. This is the first encounter that you appreciate a murmur during cardiac auscultation. You appreciate a diastolic, high pitched, blowing murmur while listening with the diaphragm in the left sternal border. There is no palpable thrill. Which of the following is the most likely diagnosis? Aortic regurgitation Aortic stenosis Mitral regurgitation Tricuspid stenosis

Correct Answer ( A ) Explanation: The murmur of aortic regurgitation is best heard with the diaphragm in the left sternal border. It can be accentuated when a patient sits and leans forward. This is a diastolic murmur, as compared to the systolic stenotic murmur of the same valve. It is often high pitched, blowing and decrescendo in nature. A widened pulse pressure is seen due to incompetence of this valve as it allows previously "pumped-out" blood to return back into the left ventricle. Common causes include rheumatic heart disease, trauma and endocarditis, and it can also be associated with Marfan's syndrome, syphilis and ankylosing spondylitis. Aortic stenosis (B) is a systolic murmur. Mitral regurgitation (C) is a pansystolic murmur best heard at the apex. Tricuspid stenosis (D) is a diastolic rumble heard over the left sternal border at the level of the fourth intercostal space. A right ventricular thrill is commonly palpated.

Which of the following statements is true regarding the condition associated with the ECG seen above? Pain is often relieved in the supine position The ECG is the most reliable diagnostic tool Thrombolytic therapy is a potential treatment Ventricular dysrhythmias are common

Correct Answer ( B ) Explanation: The ECG is representative of acute pericarditis. This may be difficult to distinguish from the ECG seen with acute MI, coronary artery spasm, or benign early repolarization. Although there is no single test that is diagnostic for pericarditis, the ECG is the most reliable diagnostic tool. It evolves through stages that occur over time. Over the first few hours to days of the illness, there is diffuse ST segment elevation with concurrent PR segment depression. In the next stages, the ST and PR segments normalize, but the T waves flatten with subsequent deep, symmetrical T wave inversion. The final stage is normalization of the ECG. The pain of pericarditis is often worsened by the supine position (A) and relieved by sitting up. Thrombolytic therapy (C) is contraindicated in pericarditis because its use may precipitate hemorrhagic cardiac tamponade. Ventricular dysrhythmias (D) are rare in pericardial disease. Those patients who do exhibit ventricular dysrhythmias should be suspected of having concomitant myocarditis. One Step Further Question: What is the most common cause of pericarditis? Reveal Answer: Rapid Review Acute Pericarditis Idiopathic > viral (Coxsackie) Pleuritic chest pain radiating to the back Pain ↓ with leaning forward Pericardial friction rub ECG: diffuse STE, PR depression NSAIDs

Which of the following diagnostic studies has the highest sensitivity to evaluate for acute radiculopathy? Cerebrospinal fluid analysis Electromyography Magnetic resonance imaging X-ray

Correct Answer ( C ) Explanation: Magnetic resonance imaging (MRI) should be performed when evaluating for acute radiculopathy because it is the most sensitive diagnostic tool for a herniated nucleus pulposus or other intraspinal pathology. Radiculopathy is a disease of a nerve root caused by compression, irritation, or swelling. It most commonly is a result of a herniated disc but can also be due to malignancy, infection, or vascular disorders. An MRI is the diagnostic test of choice because it is the least invasive and least radiation-inducing procedure to visualize the cause of radiculopathy. If a patient can't undergo an MRI because of intolerance or a contraindication, a CT scan or CT myelogram with contrast can be performed instead. Cerebrospinal fluid analysis (A) is not routinely performed in the workup of radiculopathy and is only indicated if no other causes are identified or if there is a high suspicion for neoplasm or infection. Leptomeningeal carcinomatosis, Lyme disease, and cytomegalovirus are examples of causes of radiculopathy that may only be found through testing of the cerebrospinal fluid. A lumbar puncture, however, should not be obtained when there is high suspicion for an epidural abscess because there is a high risk of introducing infection into the central nervous system if the needle passes through an infected area. Electromyography (B), also known as EMG, is used to evaluate the electrical activity produced by muscles and the nerve roots that control them. This study examines the integrity and utilization of nerve roots and can help to localize injured nerves as well as provide information as to how long the nerve has been injured. An EMG is not routinely performed and should be reserved for those whose neuroimaging findings do not correlate with the patient's symptoms or when it is important to determine how long the nerve has been damaged. X-ray (D) is not routinely performed when evaluating radiculopathy as it can only visualize bony processes and not the discs and therefore would not provide enough information to make a diagnosis. If fracture, spinal stenosis, infection, or malignancy are suspected, an X-ray should be performed to evaluate the alignment of the spine, but it should not routinely be performed.

A 60-year-old man with a history of peptic ulcer disease presents to your office with a complaint of intense pruritus after showering. Physical exam reveals splenomegaly and injection of bilateral conjunctivae. Laboratory findings include elevated hemoglobin, hematocrit, red blood cell mass and vitamin B12. Which of the following is the most likely diagnosis? Hodgkin lymphoma Hypereosinophilic syndrome Polycythemia vera Sickle cell anemia

Correct Answer ( C ) Explanation: Polycythemia vera is a bone marrow disease that causes chronic myeloproliferative neoplasms. Other diseases in this category include chronic myelogenous leukemia and essential thrombocytopenia. Patients often present with pruritus after a warm shower or bath, burning pain in hands and feet, venous or arterial thromboses, transient visual disturbances and gastrointestinal complaints. Physical exam often reveals splenomegaly, hepatomegaly, conjunctival injection, skin excoriations, and gouty tophi. Laboratory testing results in elevated hemoglobin, hematocrit, red blood cell mass and vitamin B12. Patients with polycythemia vera may have a history of peptic ulcer disease and endoscopy can show gastroduodenal erosions. Itching is a common symptom found in patients with Hodgkin lymphoma (A), but is not aquagenic like polycythemia vera. Patients with Hodgkin lymphoma generally experience asymptomatic lymphadenopathy and the disease is defined by the presence of Reed-Sternberg cells. Laboratory findings in hypereosinophilic syndrome (B) include elevated vitamin B12. Signs and symptoms of hypereosinophilic syndrome are related to end-organ damage caused by eosinophils. Sickle cell anemia (D) causes splenic dysfunction and blood testing shows decreased hemoglobin and hematocrit, along with sickled red blood cells and Howell-Jolly bodies on peripheral smear.

A previously healthy 35-year-old woman presents to your clinic with questions about breast cancer screening. She has no family history of breast cancer and wants to know when she should start screening. Per the United States Preventive Services Task Force, which of the following is the most appropriate next step in management? Initiate screening magnetic resonance imaging at age 40 Initiate screening mammography at age 40 Initiate screening mammography at age 50 Initiate screening ultrasound at age 50

Correct Answer ( C ) Explanation: Screening guidelines for cancer are developed after consideration of a number of factors including the benefits and harms of screening, scientific evidence of the risks of the condition and cost. Guidelines for breast cancer screening have more scientific evidence than any other type of cancer screening and have changed in recent years. There remain some discrepancies between different medical groups regarding recommendations for screening. The United States Preventive Services Task Force recommends screening mammography for women every two years from ages 50-74 years. Earlier and more frequent screenings may be individualized based on patient risk factors such as family history of breast cancer. Screening mammography for women may start at age 40 (B) in individualized cases based on risk factors. Magnetic resonance imaging (A) is not recommended as a diagnostic tool in breast cancer screening since there is not enough evidence to support its use. Ultrasound (D) is not used as a screening tool in breast cancer screening. Women who have suspicious lesions on mammography are generally sent for ultrasound to better visualize the lesions and determine next steps.

A 20-year-old woman presents with weakness in her left wrist. She states that she fell asleep in a chair after a night of heavy drinking. On physical examination, she is unable to extend her wrist. What nerve is compromised? Axillary nerve Median nerve Radial nerve Ulnar nerve

Correct Answer ( C ) Explanation: This patient presents with a wrist drop caused by radial neuropathy also called "Saturday Night Palsy." The radial nerve arises from the C5-T1 roots. It controls extension of the fingers, thumb, wrist and elbow. Symptoms of radial neuropathy depend on the location of compression. Compression in the axilla (typically from improper use of crutches) causes weakness of extension at the elbow, wrist and fingers. More typically, the nerve is compressed between the humeral shaft and another hard surface and results in weakness with extension at the wrist and fingers. This typically occurs from deep sleep (often secondary to inebriation). About 90% of radial nerve mononeuropathies that occur during sleep, coma or anesthesia recover within 6-8 weeks. Patients should be placed in a wrist splint with 60 degrees of dorsiflexion to prevent atrophy and contractures. The axillary nerve (A) causes weakness of the shoulder (deltoid) and decreased sensation over the shoulder. Median nerve neuropathy (B) causes numbness in the 1st, 2nd and 3rd digits. Ulnar nerve (D) neuropathy symptoms depend on location but do not cause weakness with extension at the wrist.

A 28-year-old man with a history of left renal agenesis presents to the Emergency Department with sudden onset right flank pain. Vital signs are BP 140/70 mm Hg, HR 107, RR 18, and T 98.1°F. Urinalysis is negative for nitrites and leukocyte esterase, but is 2+ for blood with 10-20 red blood cells/hpf. The patient receives a dose of analgesia which controlled his pain. An ultrasound is performed which shows marked hydronephrosis of his right kidney but does not demonstrate a renal calculus. Which of the following is the most appropriate next step in management? Discharge home with analgesics and outpatient follow up Emergency department Foley catheter placement to relieve urinary obstruction Obtain an urology consultation for emergent decompression Perform a CT scan of his abdomen and pelvis with intravenous contrast

Correct Answer ( C ) Explanation: Urinary calculi are a common condition presentation to Emergency Departments. There is a 2:1 male predominance and the first episode usually occurs between 20 and 50 years of age. The majority of renal calculi are composed of calcium and are attached to either oxalate or phosphate. Ten to fifteen percent of stones are composed of struvite (magnesium-ammonium-phosphate) which are often precipitated by urea-splitting bacteria such as proteus. Uric acid stones make up the majority of the remainder. Patients with simple non-obstructing stones are safe to discharge home with analgesics and outpatient follow up. However, patients with complicating features including intractable pain or vomiting, fever, signs of infection, have a solitary kidney or transplanted kidney with or without signs of obstruction, acute renal failure, obstructing stone with signs of urinary infection, hypercalcemic crisis, urinary extravasation, significant co-morbidities or large size of stone especially in the proximal ureter should have a urologic consultation and likely admission. Patients with a solitary kidney, complete obstruction, and sepsis often require emergent decompression. Therefore, this patient should have an urgent urology consultation. Ultrasound has a modest sensitivity and specificity for detecting renal stones but may miss stones ≤ 5 mm in size. It also helpful in diagnosing proximal and distal ureteral stones, but is insensitive for identification of mid-ureteral stones. Ultrasound is 98% sensitive for detecting hydronephrosis and can be used in patients who are not candidates for CT or IV urography, as is the case in the patient in the above clinical vignette given his solitary kidney. Discharge home with analgesics and outpatient follow up (A) is inappropriate in this patient who is at high risk for complications given his solitary kidney and and significant hydronephrosis, indicating ureteral obstruction. Given the history of renal agenesis, it is conceivable that the severe hydronephrosis could be caused by a distal obstruction, such as benign prostatic hypertrophy. However, with the patient's sudden onset flank pain and 2+ blood in the urine, a ureterolith is more likely. Placing a Foley catheter (B) will be ineffective at decompressing the patient's kidney. A Foley catheter is used to decompress bladder outlet obstruction. CT abdomen/pelvis without contrast (D) may be useful to identify the location and size of the stone/obstruction. A CT abdomen/pelvis with contrast may aide in the evaluation of renal function by the passage of contrast material through the kidneys and ureters into the bladder, however, the contrast dye will make locating and characterizing the nephroliths and ureteroliths difficult. Moreover, contrast is contraindicated in patients with acute renal failure, renal insufficiency or who are at high risk of renal complications, such as the patient in this clinical scenario.

Which of the following is the most common congenital heart defect? Atrial septal defect Coarctation of the aorta Patent ductus arteriosus Ventricular septal defect

Correct Answer ( D ) Explanation: Ventricular septal defect (VSD) is most common, responsible for 25% of all congenital heart defects. Most are small and close spontaneously over time. However, larger lesions may persist, eventually leading to pulmonary hypertension and right-sided heart failure. The classic VSD murmur is holosystolic and best heard at the left lower sternal border. Atrial septal defects (ASD) (A) are rarely recognized in the neonatal period and typically manifest during adolescence, with dyspnea on exertion or, during adulthood with atrial dysrhythmias. An ASD is associated with a fixed split S2 and a loud S1. Coarctation of the aorta (B) involves a narrowing around the ductus arteriosus. Mild cases usually present later in life with hypertension and a chest radiograph that shows rib notching. The ductus arteriosus (C) usually closes by one week of life. Rarely, it will remain open beyond this, but when it does, a continuous machine-like murmur can be heard at the left upper sternal border.

Question: What is the definitive form of treatment for infantile hypertrophic pyloric stenosis?

Question: What is the definitive form of treatment for infantile hypertrophic pyloric stenosis? Answer: Surgery (pyloromyotomy). Rapid Review Pyloric Stenosis Patient will be 2-6 weeks old Complaining of nonbilious projectile vomiting after feeding and early satiety PE will show RUQ olive-like mass (hypertrophied pylorus) Labs will show hypochloremic hypokalemic metabolic alkalosis Diagnosis is made by ultrasound or UGI series (string sign) Treatment is surgical

Question: Which type of malignancy is most associated with malignant acanthosis nigricans?

Answer: Gastric cancer. Rapid Review Acanthosis Nigricans Patient will be obese or diabetic PE will show thickened, velvety, darkly pigmented plaques on the neck or axillae Comments: Screen for diabetes in those not yet diagnosed

Question: What type of cardiomyopathy is most common in patients who consume alcohol?

Answer: Dilated cardiomyopathy. Rapid Review Atrial Fibrillation Observe when initiated by alcohol Irregularly irregular No P waves Narrow QRS unless conduction block or accessory pathway Unstable: cardioversion Stable: <48 hours duration: cardiovert to sinus rhthym >48 hours duration: rate control with CCBs, ßBs, anticoagulate, echo to r/o thrombus, then cardioversion

Question: What chromosomal abnormality is Hirschsprung disease commonly associated with?

Answer: Down syndrome (trisomy 21). Rapid Review Intussusception (Telescoping Bowel) Patient will be a child 5 months - 3 years old Complaining of colicky abdominal pain, vomiting and bloody stools (currant jelly) Diagnosis is made by ultrasound ("target sign") Most commonly caused by a tumor or meckel's diverticulum Treatment is air/contrast enema

Question: How long until the onset of action of phenobarbital?

Answer: The onset of action is 15-30 minutes. Rapid Review Status Epilepticus Seizure lasting > 5 minutes or > 2 discrete seizures without recovery of consciousness Causes: antiepileptic therapy discontinuation, medication noncompliance BZDs, phenytoin (second line), phenobarbital (third line)

Question: Infectious mononucleosis is causes by which virus?

Answer: Epstein Barr Virus (EBV). Rapid Review Infectious Mononucleosis Patient will be complaining of low-grade fever, headache, malaise, severe fatigue PE will show mildly tender lymphadenopathy involving the posterior cervical chain, hepatosplenomegaly Diagnosis is made by heterophile antibody test (monospot test), generalized maculopapular rash following administration of amoxicillin Most commonly caused by Epstein-Barr virus Treatment is self-limiting, refrain from contact sports for four weeks post-infection

Question: What are criteria for admitting a patient with cellulitis on outpatient antibiotics?

Answer: Failure to improve or worsening cellulitis after 48-72 hours of appropriate therapy. Rapid Review Cellulitis Patient will be complaining of pain, redness, swelling PE will show tenderness, erythema with poorly demarcated borders, lymphedema Most commonly caused by Staph aureus and Streptococci

Question: True or false: Toxic megacolon most commonly affects young adults (aged 20-40 years)?

Answer: False. Rapid Review Toxic Megacolon Most common cause: inflammatory bowel disease Colon dilated > 6 cm on radiographs Systemic toxcity High risk of perforation Rx: IVF, ABX, IV corticosteroids, emergent surgical consultation

Question: What is status migranosus?

Answer: A severe migraine headache that persists >72 hours. Associated symptoms are usually debilitating and hospitalization may be required. Rapid Review Migraine Headache Gradual onset, unilateral > bilateral, throbbing, pulsating First episode: < 30 years old F > M Triggers: cheese, OCPs, pregnancy, menstruation Without aura: most common, N/V, photophobia, phonophobia Aura: scotoma, flashing lights, sounds Abortive rx: triptans, DHE, antiemetics, NSAIDs Ppx: ßBs. CCBs, TCAs Triptans, DHE: contraindicated in HTN or CV disease

Question: What is the precursor lesion to the development of esophageal adenocarcinoma?

Answer: Barrett's esophagus. Rapid Review Esophageal Neoplasm Patient will be a man With a history of weight loss or chronic GERD Complaining of progressive dysphagia to solid foods Diagnosis is made by endoscopy with biopsy Most common type adenocarcinoma Comments: usually a complication of GERD/Barrett's esophagus

Question: What is sign on physical examination associated with torsion of the testicular appendage?

Answer: Blue dot sign. Rapid Review Epididymitis Patient will be complaining of gradual onset unilateral scrotal pain PE will show increased color flow on doppler, relief with testicular elevation (Prehn's sign) Most commonly caused by < 35 yo: C. trachomatis, N. gonorrhea > 35 yo: E. coli, Pseudomonas Treatment is < 35 y/o ceftriaxone/doxycycline, > 35 y/o ciprofloxacin

Question: Which of the personality disorders are associated with an increased risk of suicide attempts?

Answer: Borderline and histrionic personality disorders. Rapid Review Borderline Personality Disorder (BPD) Cluster B personality disorder F > M Splitting Unstable mood and relationships Self-mutilation, ↑ suicide risk

Question: What is the name of the spirochete that causes Lyme disease?

Answer: Borrelia burgdorferi, which is transmitted by the Ixodes tick. Rapid Review Lyme Disease NE USA Campers/hikers Ixodes tick harbors Borrelia burgdorferi Stage I: erythema migrans (pathognomonic), viral-like syndrome Stage II: arthritis, myocarditis, bilateral Bell's palsy Stage III: chronic arthritis, chronic encephalopathy Rx: doxycycline CNS/cardiac involvement: IV ceftriaxone Pregnant/children rx: amoxicillin

Question: In hypocalcemia, what do you give to stabilize the heart?

Answer: Calcium gluconate. Rapid Review DiGeorge Syndrome Patient with a history of congenital heart disease Complaining of recurrent infections PE will show hypoplasia of thymus and parathyroids Labs will show hypocalcemia CXR will show absent thymic shadow Most commonly caused by 22q11 deletion

Question: What is the medical treatment for patients with trigeminal neuralgia?

Answer: Carbamazepine (primary) or gabapentin (secondary). Rapid Review Trigeminal Neuralgia Sudden unilateral paroxysms of pain in gums, cheek, chin, temporal forehead Pain in V2 and V3 distributions, not V1 Right side > left side Triggers: chewing, brushing teeth, touching face, hot/cold exposure Carbamazepine

Question: What is the preferred antibiotic treatment for mastitis infections?

Answer: Cephalosporins or dicloxacillin are the preferred antibiotics. Rapid Review Mastitis Patient will be a breastfeeding mother Complaining of breast erythema, tenderness, fever Most commonly caused by Staph. aureus Treatment is warm compresses, dicloxacillin, amoxicillin Comments: continue breastfeeding to avoid progression to abscess

Question: What treatment is recommended for asthma classified as mild persistent?

Answer: Daily low-dose inhaled glucocorticoid.

Question: Before what day of abstinence is it rare for delirium tremens to develop?

Answer: Day three. Rapid Review Ethanol Withdrawal Autonomic hyperactivity (↑ HR, ↑ BP, diaphoresis) Visual, tactile hallucinations Seizures Delirium tremens: autonomic hyperactivity, psychosis, peaks 2-5 days after cessation Tremulousness → hallucinations → seizures → delirium tremens Rx: BZDs

Question: What is reported to be the most common ulcerative sexually transmitted disease in the US?

Answer: Genital herpes. Rapid Review Primary Syphilis Patient will have been sexually active 2 to 3 weeks ago Complaining of bump on his penis PE will show painless, "punched out" lesion Diagnosis is made by dark field microscopy Most commonly caused by Treponema pallidum Treatment is single IM injection of benzathine penicillin Comments: syphiLIS is painLESS ulcer

Question: What is the goal heart rate during an treadmill exercise stress test?

Answer: Goal= 85% of maximum predicted heart rate [220 - patient age].

Question: What clinically distinguishes hemolytic uremic syndrome (HUS) from thrombocytopenic thrombotic purpura (TTP)?

Answer: HUS causes less change in mental status and more renal dysfunction. Patients with HUS tend to be younger, and onset is associated with bacterial gastroenteritis. Rapid Review TTP/HUS HUS: E. coli O157:H7 Autoimmune hemolysis, Renal failure, Thrombocytopenia (ART) Bloody diarrhea Avoid ABX TTP: ART + Fever + Neurologic abnormalities (FAT RN) Normal coagulation studies Plasmapheresis

Question: What are the complications of otitis media?

Answer: Hearing impairment, TM perforation, labyrinthitis, facial nerve palsy, meningitis, extradural abscess, mastoiditis, and lateral venous sinus thrombosis. Rapid Review Acute Otitis Media Patient will be an infant or young child Complaining of ear pain, fever, URI symptoms PE will show TM erythema and decreased mobility of TM Most commonly caused by viral > bacterial (S. pneumoniae most common) Treatment is amoxicillin

Question: What is the most common source of septic arthritis?

Answer: Hematogenous spread. Rapid Review Arthrocentesis Contraindications: overlying skin infection, bleeding diathesis, bacteremia Ankle: medial to the anterior tibial tendon and directed toward the anterior edge of the medial malleolus. Elbow: lateral aspect distal to the lateral epicondyle and directed medially Knee: midpoint or upper portion of patella and directed beneath the posterior surface of patella into joint Shoulder: inferior and lateral to the coracoid process and directed posteriorly toward the glenoid rim

Question: Which disease is characterized by intrinsic acute renal failure, microangiopathic hemolytic anemia, and thrombocytopenia?

Answer: Hemolytic uremic syndrome (HUS).

Question: Which hepatitis is caused by a defective RNA-containing virus?

Answer: Hepatitis D (delta).

Question: What is the recommended treatment protocol for acute pyelonephritis/cyst infection in those with polycystic kidney disease?

Answer: Hospital admission, blood and urine cultures and intravenous fluoroquinolones. Rapid Review Autosomal Dominant Polycystic Kidney Disease Autosomal dominant Bilateral cysts → progressive renal failure Flank pain Hematuria HTN → berry aneurysm, intracerebral hemorrhage BP control: ACEIs, ARBs Definitive rx: kidney transplantation

Question: What is the most concerning complication of a newborn born to a diabetic mother?

Answer: Hypoglycemia. Rapid Review Gestational Diabetes Mellitus Screening between 24 and 28 weeks with 50 g glucose load (abnormal: glucose >130 mg/dL after 1 hr) Diagnostic criteria: Random glucose > 200 mg/DL on 2 separate occasions or Fasting glucose >126 mg/dL on 2 separate occasions or A 100 g glucose challenge with >95 mg/dL fasting, 180 mg/dL at 1 hour, >155 mg/dL at 2 hours, or >140 mg/dL at 3 hours Rx: lifestyle changes, fetal growth monitoring, insulin Fetal risks: macrosomia, respiratory distress syndrome, neonatal hypoglycemia ↑ Maternal risk of type 2 DM

Question: What are the unique laboratory findings of pleural effusions associated with rheumatoid arthritis?

Answer: Low glucose, low pH and high LDH. Rapid Review Rheumatoid Arthritis Women in 20s/30s AI destruction of synovial joints Morning stiffness lasting > 30 minutes MCP, PIP Bilateral ulnar deviation at MCP Baker's cyst Systemic sx Lung: interstitial fibrosis, effusions Spine: atlantoaxial joint subluxation; cord/vertebral artery compression Anemia NSAIDs

Question: Which heparin should be avoided in patients with severe renal insufficiency?

Answer: Low molecular weight heparin is almost exclusively renally cleared and should be avoided with creatinine clearance < 30 mL/min. Rapid Review Deep Vein Thrombosis (DVT) Patient with a history of smoking, long distance travel, surgery, oral contraceptives use Complaining of unilateral leg edema, leg pain, tenderness and warmth PE will show positive Homan's sign Diagnosis is made by first ultrasound, Gold Standard: venography Most commonly caused by stasis, hypercoagulable state, trauma (Virchow's triad) Treatment is IV heparin and switch to warfarin Comments: Risk stratification by Well's criteria

Question: Which condition is associated with bilateral central vision loss and is the leading cause of permanent blindness in the elderly?

Answer: Macular degeneration. Rapid Review Cataract Opacity of lens Painless blurred vision Treatment is surgical

Question: What is the treatment for multifocal atrial tachycardia?

Answer: Most commonly, treat the underlying cause which will often improve the dysrhythmia. Nodal agents are not as effective as in other dysrhythmias. Rapid Review Multifocal Atrial Tachycardia Irregularly irregular ≥ 3 different P waves Rate: 100-180 Nonconducted P waves are present Pulmonary disease/COPD Treat underlying condition Symptomatic: CCBs for rate control

Question: Are QRS complexes dropped in first degree heart block?

Answer: No, P wave conduction is delayed but not actually blocked, so no QRS complexes are dropped. Rapid Review Heart Block: First-Degree Delayed AV node conduction PR interval > 0.2 seconds No specific rx

Question: What population has a high fatality rate if infected with Hepatitis E?

Answer: Pregnant women during the second and third trimesters. Rapid Review Norovirus Patient with a history of recent travel on a cruise ship Complaining of nausea, vomiting, and diarrhea Treatment is supportive Comments: most common cause of viral gastroenteritis

Question: What is the recommended initial treatment in patients with severe constipation?

Answer: Suppositories.

Question: High ankle sprains involve a partial tear of which ligament?

Answer: Syndesmosis. Rapid Review Ankle Sprain Partial or complete tearing of ≥ 1 ligaments Most often due to inversion mechanism Anterior talofibular ligament (ATFL) most commonly sprained Grade I: microscopic tear, Grade II: incomplete tear, Grade III: complete tear RICE

Question: What is a Jarisch-Herxheimer reaction?

Answer: The onset of fever, myalgias, headache, tachycardia and tachypnea after initiation of antibacterial treatment of a spirochete illness. Rapid Review Lyme Disease NE USA Campers/hikers Ixodes tick harbors Borrelia burgdorferi Stage I: erythema migrans (pathognomonic), viral-like syndrome Stage II: arthritis, myocarditis, bilateral CN7 palsy Stage III: chronic arthritis, chronic encephalopathy Rx: doxycycline CNS/cardiac involvement: IV ceftriaxone Pregnant/children rx: amoxicillin

Question: Which coronary artery most commonly supplies the inferior wall with blood?

Answer: The right coronary artery. Rapid Review Inferior ST-Elevation Myocardial Infarction RCA occlusion ST elevations: II,III, aVF RV infarction ST elevations: V4R and V5R JVD, hypotension Rx: IVF NTG contraindicated

Question: A focused diabetic physical examination must include, at minimum, evaluation of which areas?

Answer: Vital signs (autonomic neuropathy), fundoscopy (retinopathy) and feet (diminished pulses, neuropathy). Rapid Review Diabetes Mellitus Type 1 Autoimmune destruction of β cells <30 years old Polyuria, polydipsia, weight loss DKA Insulin always necessary

Question: Can succinylcholine be used for rapid sequence intubation in an acute burn patient?

Answer: Yes. The concerning change in muscle receptors that occurs from burns takes place over 7-10 days after the burn. Rapid Review Thermal Burns Fire: obtain CO levels Industrial or house fire: suspect CN toxicity Superficial: similar to sunburn Superficial partial: red, painful, blisters Deep partial: white, leathery, relatively painless Full-thickness: charred, insensate Rule of 9s Parkland formula: 4 mL/kg × % total BSA burned (pediatrics give 3 mL/kg); 50% given in first 8 hours, remainder over 16 hours Target urine output: 0.5-1 mL/kg/h in adults; 1-2 mL/kg/h in children Consider escharotomy for circumferential and full-thickness burns

A 32-year old woman and her 2 children present to your office with a 4-day history of intense itching that is worse at night. They specifically complain of pruritus under their arms and in between their fingers and toes. They deny any changes in household cleaners, new carpets or any allergies to pets. On physical exam you notice small papules and vesicles on the extremities and the axilla. You also notice burrows in between the fingers and toes. Skin scrapings confirm your suspected diagnosis. Which of the following is the first-line treatment for this disease? Corticosteroid cream Malathion lotion Permethrin cream Topical mupirocin

Correct Answer ( C ) Explanation: Based on the history and physical exam, this patient and her children most likely have scabies. Human scabies is an intensely pruritic skin infestation caused by the host-specific mite Sarcoptes scabiei hominis. Bite distribution and intractable pruritus that is worse at night, as well as scabies symptoms in close household contacts should immediately rank scabies at the top of the clinical differential diagnosis. Small papules, vesicles and burrows in the webbed spaces of the fingers and toes, axilla, elbow and belt line are the classical distribution of scabies lesions. The treatment of choice for primary scabies infection is the application of topical scabicidal agents, with repeat application in 7 days. The treatment of choice is permethrin 5% lotion. Individuals affected by scabies should avoid skin-to-skin contact with others. Patients with typical scabies may return to school or work 24 hours after the first treatment. Secondary infections may require antibiotics such as topical mupirocin (D). Patients who have had scabies misdiagnosed often times are treated with topical corticosteroids (A). This delays the correct diagnosis and can potentially worsen the outcome. A 0.5 percent malathion lotion (B) is available for the treatment of head lice, not scabies.

A mother brings her 7-year-old boy into the clinic. She reports that the child has been complaining of fatigue, headache and myalgias since he returned home from a camping trip with his father. His mother points out an annular rash with central clearing on the child's torso. Which of the following is the most appropriate treatment for this patient's condition? Amoxicillin Azithromycin Doxycycline Erythromycin

Correct Answer ( A ) Explanation: Amoxicillin is the first line treatment of Lyme disease in children. Early disease usually occurs a few days to one month after an Ixodes tick bite and is typically characterized by an erythema migrans skin lesion with or without nonspecific complaints resembling a viral syndrome. For adults and children ≥ 8 years of age with early Lyme disease, doxycycline, amoxicillin, or cefuroxime is recommended. Doxycycline should be avoided in children < 8 years of age and pregnant or lactating women. Such patients should receive amoxicillin. Doxycycline (C) is often used because it is effective in treating a potential coinfecting agent and has the best penetration into the central nervous system; however, it should be avoided in children under the age of eight years. In the United States, macrolides such as azithromycin (B) and erythromycin (D) are not recommended as first-line therapy for patients with erythema migrans because they are less effective than amoxicillin.

An 8-year-old boy presents with a complaint of excessive thirst and urination. His mother comments that he sleeps "all the time". Which of the following test result scenarios is needed to make a diagnosis of type 1 diabetes mellitus? Fingerstick glucose >126 mg/dL, taken between 5am and 7am Serum hemoglobin-A1c of 6.1%, drawn upon first presentation of polyuria, polydipsia or polyphagia Two-hour serum glucose >200 mg/dL, checked after a 75g oral glucose tolerance test Urine ketones >2 mg/dL, sampled once between the ages of 5 and 10 years

Correct Answer ( C ) Explanation: Diabetes mellitus type 1 (DM1) is a condition of glucose dysregulation and hyperglycemia. It is due to an autoimmune attack on the pancreatic beta-cells. Onset usually occurs in childhood, but can also occur in adults aged 30-40 years. Symptoms include polyuria, polydipsia, polyphagia, unexplained weight loss, fatigue and blurred vision. Some patients present with diabetic ketoacidosis. The American Diabetic Association diagnostic criteria are as follows: a fasting serum glucose >126 mg/dL, or a 2-hour serum glucose >200 mg/dL post 75g oral glucose tolerance test, or a random serum glucose >200 mg/dL in a patient with the above symptoms. Screening is recommended in children of first-degree relatives with DM1, before the age of 10 years and once in adolescence. This screening is performed via an assay that detects the presence of anti-islet antibodies. Treatment includes self-monitoring, diet and exercise modifications and lifelong insulin therapy. In order to make a diagnosis, any fingerstick glucose test (A) must be confirmed with a serum test. The ADA recommends a serum hemoglobin-A1c (B) test only if DM1 is suspected in a patient who displays none of the classic symptoms. The above patient presents with the classic symptoms. The ADA recommends using this test as a monitoring measure instead of a diagnostic measure, although some providers use it for diagnosis. To make a diagnosis, two measurements >6.5% are necessary. Pre-diabetes can be considered if the hemoglobin-A1c is between 5.7 and 6.4%, as in the above patient. The presence of urine ketones (D) is not a reliable marker for diagnosing DM1 or DKA. A more reliable indicator of DKA is the serum acetone level.

Which of the following is contraindicated in children with bloody diarrhea and no confirmatory infection by stool culture? Antibiotics Blood transfusion Hemodialysis Intravenous fluid hydration with normal saline

Correct Answer ( A ) Explanation: Antibiotics and antidiarrheal agents should be avoided in children who have hemorrhagic diarrhea because both agents may increase subsequent risk of developing HUS. Hemolytic uremic syndrome (HUS) is characterized by the triad of microangiopathic hemolytic anemia, thrombocytopenia, and acute renal failure. HUS is the leading cause of renal failure in children in the United States. HUS is characterized by a prodrome of abdominal pain and bloody diarrhea. It develops 2-14 days after the start of diarrhea. It is associated with pallor with a petechial or purpural rash, decreased urine output, and rarely CNS symptoms. HUS is most commonly caused by Shiga toxin producing E. coli; one well-known serotype is O157:H7. The CDC recommends that all patients with bloody diarrhea be tested for E. coli O157:H7 as part of the standard stool culture. Approximately 80% of children with HUS will require a blood transfusion to avoid cardiac or pulmonary compromise (B) with a recommended posttransfusion goal of 8 to 9 g/dL. Up to 50% of patients with HUS require dialysis during the acute phase of the illness (C), although the prognosis is favorable. Intravenous fluids (D) are reasonable, as most patients are intravascularly volume depleted.

A 19-year-old man presents with headache and a peripheral cranial nerve seven palsy. He states that he was recently hiking in Connecticut and had numerous tick bites. What CSF finding is most sensitive for Lyme meningitis? Borrelia burgdorferi antibody Decreased glucose level Decreased protein level Positive PCR assay

Correct Answer ( A ) Explanation: B. burgdorferi antibody is the most sensitive test for Lyme meningitis and will be present in 80-90% of patients. Lyme disease is the most common tick borne illness in the US. The disease is caused by Borrelia burgdorferi, a spirochete. After a bite from the Ixodes scapularis tick and transmission of the spirochete, patients typically develop a non-specific viral illness accompanied by the erythema migrans rash. The rash appears at the site of the tick bite and spreads outwards eventually with central clearing. Early disease can progress to the acute disseminated form where hematogenous spread can cause multiorgan involvement. Neurologic signs develop about 4 weeks after the initial infection. The most common presentation of neurologic involvement is a fluctuating meningoencephalitis with accompanying cranial and peripheral neuropathies. The most common cranial neuropathy is a seventh nerve palsy. CSF samples should be obtained and sent for B. burgdorferi antibodies (IgG or IgA) since this is the most sensitive test. CSF glucose (B) levels are typically normal in Lyme and protein (C) is usually elevated. Neither of these findings is sensitive for diagnosing Lyme meningitis. CSF PCR (D) is positive in less than half of patients.

What is the most common cause of lens opacity? Cataract Glaucoma Hyphema Optic neuritis

Correct Answer ( A ) Explanation: Nuclear sclerosis cataract is the most common cause of lens opacity. Cataracts are common and, as a result, cataract surgery is the most common U.S. surgery performed. A cataract causes opacity to the lens that decreases visual acuity. This decrease in acuity occurs from the deposition of insoluble proteins in an otherwise transparent tissue. The most common complaint associated with cataracts is painless blurred vision with an exam that may show visible opacities or a diminished red reflex, or both. The decision for cataract surgery with intraocular lens implantation depends on the degree of vision loss and the daily requirements of the patient. In patients with cataracts, indications for surgery include the patient's preference and needs, functional disability by Snellen visual acuity test and visual field testing, and concomitant ocular problems. Glaucoma (B) is a disorder characterized by increased ocular pressures leading to damage to the optic nerve and loss of peripheral vision. Hyphema (C) is usually associated with trauma or elevated intraocular pressures and causes a layering of blood in the anterior chamber. Optic neuritis (D) is localized inflammation of the optic nerve sheath, resulting in reduced neuronal transmission and decreased visual acuity. Generally, there is a loss of color vision and red desaturation noticed by the patients.

A newly diagnosed type 1 diabetic complains of severe dyspnea. The patient is diagnosed with acute bacterial pneumonia, admitted to the hospital, and started on antibiotic therapy. During the second day of admission, he develops mental status changes, increased urinary output, vomiting and abdominal pain. You notice deep and labored breathing on examination. Laboratory evaluation shows declining renal function and elevated levels of ß-OH-butyrate. Arterial blood gas evaluation reveals an elevated anion-gap, pH of 7.18 and bicarbonate of 20 mmol/L. Which of the following is the most appropriate management at this time? Insulin and intravenous fluids Nephrology consultation Pulmonology and infectious Disease consultation Sublingual glucose and intravenous glucagon

Correct Answer ( A ) Explanation: The patient is suffering from diabetic ketoacidosis (DKA), which is most commonly seen as a complication of type 1 diabetes, but also can occur in ketosis-prone type 2 diabetes. Precipitants include infection, exogenous steroids, failure to take prescribed insulin, inflammation, ischemia, intoxication and infarction. In the setting of insulin deficiency and depleted hepatic glycogen stores, fatty acids become the energy metabolism substrate of choice. This metabolism shift in the liver causes more ketone production, and as a result, increased serum and urine ketosis, namely elevated levels of acetoacetate and ß-OH-butyrate. Symptoms include polyuria, polydipsia, dehydration, nausea and vomiting, Kussmaul respirations (deep, labored breathing) and mental status changes. Laboratory changes include hyperglycemia, early increased anion-gap metabolic acidosis, late nonanion gap metabolic acidosis, elevated urine and serum ketones, elevated BUN and Cr, sodium and potassium depletion, leukocytosis and elevated amylase. Treatment includes evaluating for precipitants, aggressive hydration, intravenous insulin, laboratory monitoring and electrolyte replacement. The declining kidney function is most likely as result of dehydration. Recognizing and treating DKA should occur before considering a nephrology consultation (B) in this setting. Once the acidosis and hyperglycemia respond, it may be prudent to ask nephrology for help in restoring electrolytes, normalizing renal function and setting up diabetic outpatient renal monitoring. On the same note, the above clinical picture is not clearly related to underlying pulmonary disease or improper choice of antibiotic. As such, specialist consultation (C) should be considered after correcting DKA. Sublingual glucose and intravenous glucagon (D) are used in treating a hypoglycemic, not hyperglycemic, episode.

Which of the following clinical scenarios in a patient with chronic ethanol use should prompt admission to the hospital? Fever, tachycardia, hypertension Intoxication with vomiting Mild tachycardia, tongue fasciculations Normal vital signs, one seizure six hours ago

Correct Answer ( A ) Explanation: The spectrum of illnesses related to chronic alcohol use is quite broad and frequently encountered in the ED. Acutely intoxicated patients are common in urban settings and require monitoring for clinical sobriety and safety prior to discharge. Patients may request admission for detoxification in the setting of cessation of alcohol use. When alcohol use is abruptly stopped or markedly decreased, patients may develop alcohol withdrawal with mild symptoms, alcohol related seizures or in the most serious and life-threatening form of withdrawal, delirium tremens. The patient described here has several abnormal vital signs (fever, tachycardia, hypertension). These abnormalities are concerning for major alcohol withdrawal which is a constellation of symptoms which may include anxiety, irritability, tremors, tachycardia, fever, hypertension, decreased seizure threshold and both auditory and visual hallucinations. In its most severe form, patients develop delirium tremens, which is a severe hyper-adrenergic state with confusion, hallucinations and hemodynamic instability. This condition is life-threatening and requires aggressive treatment with benzodiazepines and possibly antipsychotics. Intoxication with vomiting (B) is a common presentation in both chronic alcohol users and binge drinkers. It is important to recognize that alcohol intoxication is a diagnosis of exclusion as the cause of a patient's altered mental status. Routine investigations include fingerstick glucose, careful history and an assessment for trauma. In most cases of intoxication, patients are monitored for clinical sobriety or metabolization of the ethanol intoxication. Attention is paid to the ability of a patient to maintain an adequate airway, especially in the context of vomiting. A patient with mild tachycardia and tongue fasciculations (C) is consistent with mild alcohol withdrawal. Patients may also develop nausea, a coarse tremor, insomnia and some hypertension. With treatment and observation for 4 to 6 hours in the ED, patients may be eligible for discharge. A patient with normal vital signs and a seizure 6 hours ago (D) is unlikely to have seizures related to severe alcohol withdrawal. Alcohol-related seizures are common. First time seizures should have the usual seizure evaluation including measurement of electrolytes and neuroimaging. If a patient had an isolated seizure and is seizure-free for a period of 6 hours without signs of ongoing withdrawal, they are eligible for discharge from the ED.

A 43-year-old man comes to the clinic complaining of a 3-week history of a "burning" sensation in his chest and nocturnal cough. He says he wakes up several times at night coughing and wheezing. He weighs 350-lbs and has a BMI of 48. If this patient's condition is left untreated, which of the following complications can most likely develop? Esophageal adenocarcinoma Hepatocellular carcinoma Small cell carcinoma of the lung Squamous cell carcinoma of the esophagus

Correct Answer ( A ) Explanation: This patient most likely has gastroesophageal reflux disease (GERD). GERD typically presents with heartburn, regurgitation, non-cardiogenic chest pain, and nocturnal cough. Complications from longstanding GERD can result in reflux esophagitis, esophageal strictures, Barrett's esophagus (intestinal metaplasia), and esophageal adenocarcinoma. The vast majority of esophageal adenocarcinomas arise from a region of Barrett's metaplasia, which is due to GERD. Additionally, in patients who eventually develop Barrett's esophagus, their risk of developing esophageal adenocarcinoma increases to at least 30-fold above the general population. Hepatocellular carcinoma (B) and small cell carcinoma of the lung (D) are not associated complications of longstanding GERD. Squamous cell carcinoma (SCC) of the esophagus (D) typically results from smoking, alcohol consumption, and diets that are low in fruits and vegetables. It occurs in the middle portion of the esophagus, whereas, esophageal adenocarcinoma most commonly occurs at the more distal portion (at the gastroesophageal junction). Globally, SCC is more common, whereas, esophageal adenocarcinoma is more common in the United States (and other Western countries).

A 74-year-old man presents to the ED with chest pain radiating to the jaw and dyspnea. His past medical history is significant for hypercholesterolemia, hypertension and diabetes. He denies illicit drug use. His blood pressure is 210/122 mm Hg. Physical exam and chest X-ray are normal. His ECG is consistent with left ventricular hypertrophy. Which of the following is the most likely diagnosis? Autonomic dysreflexia Hypertensive emergency Hypertensive urgency Sympathetic crisis

Correct Answer ( B ) Explanation: A hypertensive emergency is a severe elevation in blood pressure with evidence of end-organ damage. This requires immediate lowering of blood pressure. There is no specific blood pressure at which hypertensive emergency occurs, however, end-organ damage is less likely if the diastolic BP is < 130 mm Hg. With that being said, the well-accepted criteria for hypertensive crisis are systolic pressure ≥180 mm Hg or diastolic pressure ≥ 110 mm Hg. One must further consider the patient's baseline blood pressure, as a patient with chronic hypertension may not have end-organ damage with pressures around 200/150 mm Hg. Precipitants of hypertensive emergencies include progression of essential hypertension (especially if there is medical noncompliance), progression of renovascular disease, acute cardiac or cerebral ischemic injury and undiagnosed or progressive endocrinopathies. Symptoms of hypertensive emergency include chest pain, dyspnea and neurologic deficits. Associated clinical scenarios include encephalopathy, hemorrhagic or ischemic stroke, aortic dissection, acute myocardial infarction, acute coronary syndrome, acute renal failure, pulmonary edema with respiratory failure, microangiopathic hemolytic anemia and pre-eclampsia/eclampsia/HELLP syndrome. Autonomic dysreflexia (A) occurs in patients with spinal cord injury at T6 or above. A sensory stimuli, such as a urinary tract infection, caudal to the level of injury precipitates reflex vasoconstriction with resultant hypertension and bradycardia. This patient does not have spinal cord injury. Hypertensive urgency (C) was defined as a severely elevated blood pressure without evidence of end-organ damage. However, hypertensive urgency is a term no longer used. Sympathetic crisis (D) is associated with illicit drug use (cocaine, amphetamines and PCP). It is a rare condition which may also present, although infrequently, with pheochromocytoma, abrupt cessation of antihypertensives or high tyramine diet in patients on monoamine oxidase inhibitors.

A 19-year-old Hispanic man with a body mass index of 32 kg/m2 presents to your office with a complaint of darkened skin. Physical exam reveals thickened, velvety, darkly pigmented plaques on the back of his neck. Which of the following is the most appropriate next step in management? Referral to gastroenterology Screening for diabetes mellitus Skin biopsy Trial of topical retinoids

Correct Answer ( B ) Explanation: Acanthosis nigricans is a dermatologic condition characterized by darkened plaques, most commonly in the intertriginous sites such as the axillae or neck. A significant percentage of patients who are obese or have diabetes have this condition, and the presence of acanthosis nigricans in children is a risk factor for the development of diabetes. Ethnic differences are seen with a higher percentage of African-American and Hispanic patients having the condition than non-Hispanic whites or Asians. Rarely, acanthosis nigricans can develop as a result of malignancy. Diagnosis is by clinical examination and assessment for other disorders, including screening for diabetes mellitus, is an important next step in the evaluation of patients with this condition. Acanthosis nigricans is a benign condition and treatment is often based on cosmetic concerns. Treatment of the underlying condition is the preferred method of management. Older, non-obese adults with new onset acanthosis nigricans should be referred to gastroenterology (A) because of the risk of malignancy, most often due to tumors in the gastrointestinal tract. This is done after ruling out more common metabolic conditions. Skin biopsy (C) may be done if the clinical diagnosis is difficult to determine. There is no definitive treatment for acanthosis nigricans outside of treating the underlying condition. Topical medications including retinoids (D) have been effective in some cases of acanthosis nigricans, but are not the first step in management.

Which of the following describes a burn that causes pressure and discomfort, extends into the dermis, and may have thick-walled blisters or be leathery white? First-degree burn Second-degree deep partial thickness burn Second-degree superficial partial thickness burn Third-degree burn

Correct Answer ( B ) Explanation: Burn classification is based on burn depth. Second-degree burns are classified into superficial and deep partial-thickness burns. Deep partial-thickness burns extend into the reticular dermis. Skin color is usually a mixture of red and blanched white, and capillary refill is slow. Blisters are thick-walled and commonly ruptured and the skin may appear leathery white. Two-point discrimination may be diminished, but pressure and pinprick applied to the burned skin can be felt. Superficial partial-thickness burns usually re-epithelialize 7-10 days after injury, so the risk of hypertrophic scarring is very small. For deep partial-thickness burns, tissue may undergo spontaneous epithelialization from the few viable epithelial appendages at this deepest layer of dermis and heal within 3-6 weeks. Because these burns have less capacity for re-epithelializing, a greater potential for hypertrophic scar formation exists. In deep partial-thickness burns, treatment with topical antimicrobial dressings is necessary to prevent infection as the burn wound heals. Contraction across joints, with resulting limitation in range of motion, is a common sequela. Splash scalds often cause second-degree burns. First-degree burns (A) involve only the epidermis and are erythematous and painful, without blisters. They are usually described as looking like a sunburn. These are not considered in the calculation of total body surface area when calculating burn size. Second-degree superficial partial-thickness burns (C) are erythematous and have thin-walled fluid-filled blisters. These usually heal in 2 to 3 weeks without scarring. Third-degree burns (D) involve all layers of the dermis. The skin is firm, white, or charred and often described as leathery. This represents complete tissue destruction, and surgery is necessary except in the smallest of third-degree burns. Fourth-degree burns extend to deeper tissues, including subcutaneous fat, muscle, and bone. Significant debridement and reconstruction are required.

A 70-year-old woman presents to your office with a complaint of difficulty having bowel movements. She tells you that she has a bowel movement every 2-3 days, the stool is hard and she has to strain. Which of the following is the most appropriate initial therapy? Docusate sodium Methylcellulose Polyethylene glycol Senna

Correct Answer ( B ) Explanation: Constipation is a common complaint seen in the primary care setting and has a number of possible etiologies. Patients should be evaluated for systemic disorders or medications that may be causing the complaint. Once secondary causes have been ruled out, constipation of unknown etiology may be due to colonic transit time, pelvic floor dysfunction or both. Initial management of constipation involves educating the patient, increasing fluid intake and dietary fiber, and bulk-forming laxatives such as methylcellulose or psyllium. Increased fiber may cause bloating and flatulence. Patients should be advised to slowly increase their dietary fiber in order to improve efficacy and compliance. Patients who do not respond to initial management strategies may require other types of laxatives and patients with severe constipation require a different approach. Stool softeners such as docusate sodium (A) have few side effects, but are not as effective as other laxatives. Polyethylene glycol (C) is an osmotic agent that causes intestinal water secretion, increasing stool frequency. Care must be taken in patients with renal or cardiac disease as excessive use of osmotic agents may cause electrolyte and volume overload. Senna (D) is a stimulant laxative that increases intestinal motor activity. Stimulant agents should be used with caution in the treatment of chronic constipation as continued use may cause salt overload, hypokalemia or protein-losing enteropathy.

A 32-year-old man presents to the office with a rash on both of his hands. The rash appeared a couple days after completing yard work. He wore gloves while working and reports no similar rashes in household contacts. There are pink patches with multiple tapioca-like vesicles involving primarily the palms and web spaces on both hands. Which of the following is the most likely diagnosis? Contact dermatitis Dyshidrotic eczematous dermatitis Rhus dermatitis Scabies

Correct Answer ( B ) Explanation: Dyshidrotic eczema is an intensely pruritic, chronic recurrent dermatitis, typically involving the palms and soles. It starts as an episode of intense itching, followed by the formation of small vesicles. The vesicles are described as tapioca-like on the lateral aspect of the fingers and show confluence. Desquamation occurs over 1-2 weeks, leaving fissures and erosions. Treatment includes a high-potency topical steroid and prevention of secondary infection. Contact dermatitis (A) would be a reasonable choice if the patient had not been wearing gloves. Contact dermatitis is most often diagnosed based on the distribution of the rash and a history of exposure to irritants or allergens. Rhus dermatitis (C), also known as toxicodendron dermatitis, is a specific type of contact dermatitis to oil produced by plants in the genus Toxicodendron (poison ivy, poison oak and poison sumac) among others. Scabies (D) is a parasitic skin infection caused by the mite Sarcoptes scabiei. Scabies is a pruritic rash that can affect areas of the body with skin folds, such as the web spaces of the hands. Superficial burrows are created as the adult mite excavates through the epidermis. These burrows are accompanied by mosquito-like "bites."

A 29-year-old man presents with gradually worsening right testicular pain over the last four days. He is sexually active. On examination, his cremasteric reflex is normal. The testicle has a normal lie. There is tenderness focally over the epididymis. Which of the following test should be performed next? Epididymal aspiration Gonorrhea and chlamydia swab Rectal exam Urine wet mount

Correct Answer ( B ) Explanation: Epididymitis is the most common inflammatory condition within the scrotum. The epididymis is a tubular structure on the posterior aspect of the testicle where sperm mature. The epididymis becomes infected when retrograde transit of an infection travels from the vas deferens. In men who are sexually active, the organisms of sexually transmitted infections are likely etiologies. For these patients, testing for Gonorrhea and Chlamydia is indicated. For those not sexually active urinary pathogens become more common and a urine culture is indicated. Patients develop gradually increasing pain in the testicle and classically present later for evaluation than patients with torsion who experience severe onset of pain immediately. Men with epididymitis may complain of lower abdominal or flank pain due to inflammation of the vas deferens. Up to 75% of patients have fever. Pain begins in the epididymis and then spreads to the entire testicle. Over time, the ipsilateral hemiscrotum becomes edematous, erythematous and tender. Improvement of pain with elevation of the testicle (Prehn's sign) is often describes as characteristic for epididymitis although it is neither sensitive nor specific. If the clinical history and examination are not clear, an ultrasound of the scrotum will show hypervascularity of the affected epididymis (and testicle if it is involved). Antibiotic therapy is aimed at the most likely organism and recommended for 10-14 days. Complications of epididymitis include orchitis and testicular abscess. Epididymal aspiration (A) has been described as a mechanism to identify the causative organism of epididymitis. However, this has no role in the ED or office setting and may be performed later for those failing treatment and no positive urine or sexually transmitted infection testing. A rectal exam (C) is usually obtained to evaluate for bleeding or to assess for tenderness related to prostatitis. Patients with prostatitis typically present with fever, myalgias, urinary symptoms and perineal pain. In this case, the patient's clinical symptoms are consistent with epididymitis, has no rectal bleeding, and a rectal exam is not necessary. A urine wet mount (D) is not a routine test. A vaginal wet mount is performed to diagnose a yeast infection. A sample of vaginal discharge is placed on a slide and mixed with a salt solution. Under microscope examination, the hyphae of yeast are identified.

Which of the following describes a grade 3 ankle sprain? Complete ligamentous rupture with concomitant distal fibular fracture Complete ligamentous rupture with considerable swelling, pain, and significant laxity Partial tear with mild laxity and moderate pain, tenderness, and instability Partial tear without laxity and only mild edema

Correct Answer ( B ) Explanation: Grade 3 sprain is a complete rupture resulting in considerable swelling, increased pain, significant laxity, and often an unstable joint. Ankle sprain involves stretching or tearing of the ligaments of the ankle. There are three grades of ankle sprain as determined by the extent of ligamentous injury. This injury is a common cause of morbidity in the general population, and the ankle is the most commonly injured joint complex among athletes. Patients who play sports experience approximately one ankle sprain for every 1000 person-days of competition. It is estimated that more than 23,000 ankle sprains require medical care in the United States per day. Eighty-five percent of all ankle sprains occur on the lateral aspect of the ankle, involving the anterior talofibular ligament and calcaneofibular ligament. There are only 3 grades of ankle sprains and complete ligamentous rupture with concomitant distal fibular fracture (A) would be a fracture and not a sprain. Grade 2 ankle sprain (C) is a partial tear with mild laxity and moderate pain, tenderness, and instability. Grade 1 (D) is a partial tear without laxity and only mild edema.

You have made a new diagnosis of polycystic kidney disease in one of your primary care patients. Proper maintenance of normal blood pressure should be obtained with which of the following medications? Furosemide Losartan Metoprolol Verapamil

Correct Answer ( B ) Explanation: In a patient with polycystic kidney disease, the development of hypertension signifies disease progression. Therefore, it is clinically necessary to maintain proper blood pressure in these patients. Target levels for adults are 140/90 mmHg or below the seventy-fifth percentile for children. Angiotensin-converting-enzyme inhibitors (ACEi) or angiotensin II receptor blockers (ARBs) are the preferred therapeutics. Studies have shown that treatment with these drugs, as compared to other antihypertensives, is associated with preservation of normal renal function in those with PKD. Common ARBs are medications ending in -sartan, such as losartan, irbesartan and valsartan. Beta-blockers (metoprolol (C)), diuretics (furosemide (A)), and calcium-channel blockers (verapamil (D)) are used in managing hypertension, but are not the recommended class for those with PKD. If considering diuretic therapy, the thiazides, like hydrochlorothiazide, are recommended over the loop diuretics (furosemide).

A 35-year-old woman with a known history of seizure disorder is actively seizing in the ED. Which of the following is the first-line medication and route to treat her seizure? Intramuscular fosphenytoin Intravenous midazolam Oral lorazepam Rectal diazepam

Correct Answer ( B ) Explanation: In an actively seizing patient, attention is always directed to the airway first. A patient having a generalized tonic-clonic seizure has a suppressed gag reflex and is prone to aspiration of gastric contents. Therefore, patients should be placed in the left lateral decubitus position. First-line pharmacologic management for an actively seizing patient is a parenteral benzodiazepine. Benzodiazepines directly enhance GABA-mediated neuronal inhibition, affect clinical and electrical manifestations of seizures, and are highly effective at terminating seizure activity. Benzodiazepines have been shown to be more effective than phenytoin at terminating status epilepticus. They are as effective as parenteral phenobarbital, but phenobarbital is associated with a higher risk of hypoventilation and hypotension. The intravenous route is the preferred route to administer an antiepileptic because this has the quickest onset of action. Therefore, intravenous midazolam is the preferred agent from the choices listed above. Midazolam's onset of action is within 1 minute. In addition to the intravenous preparation, it is available in both intranasal and buccal formulations. It also has the least cardiovascular effects among benzodiazepines. Fosphenytoin (A) is a water-soluble prodrug form of phenytoin. Unlike phenytoin, fosphenytoin can be administered intramuscularly. Although fosphenytoin can be infused more rapidly than phenytoin can, the time to a therapeutic concentration of the active drug is the same as for intravenous phenytoin. Lorazepam (C) is considered a first-line agent for an actively seizing patient and is the most popular agent used. However, oral medications are contraindicated in an actively seizing patient due to the prolonged onset of action and risk of aspiration. Diazepam (D) is also a first-line agent for an actively seizing patient and is most commonly used as a rectal preparation in the pediatric population where intravenous access is not readily available.

A woman in her second trimester of pregnancy develops a left lower extremity deep vein thrombosis. Which of the following is the most appropriate choice of anticoagulation for this patient? Apixaban Low molecular weight heparin Unfractionated heparin Warfarin

Correct Answer ( B ) Explanation: Low molecular weight heparin is recommended at this stage of this woman's pregnancy for prevention of thromboembolism from her deep vein thrombosis. Pregnancy is a hypercoaguable state, and anticoagulation during pregnancy is indicated for women with atrial fibrillation, mechanical valve prosthesis, antiphospholipid antibody syndrome and venous thromboembolism. Unfractionated heparin, and low molecular weight heparin are approved for use during pregnancy. In contrast to anticoagulation of non-pregnant women, the choice of anticoagulant during pregnancy needs to take into account fetal safety and maternal peripartum issues such as unpredictable onset of labor. Heparins are used for most pregnant women because they do not cross the placenta and do not result in fetal anticoagulation. Low molecular weight heparin is recommended over unfractionated for all but the final weeks of the pregnancy, because they are effective and easier to administer than unfractionated heparin. Low molecular weight heparin produces a more predictable anticoagulant response than unfractionated heparin and do not require routine monitoring. Unfractionated heparin is a reasonable alternative to a low molecular weight heparin when cost or need for rapid reversal is important such as for delivery. Apixaban (A) is an oral factor Xa inhibitor. This and other newer anticoagulation agents are avoided during pregnancy because of absence of information on efficacy and fetal safety. Warfarin (D) is avoided during pregnancy because it crosses the placenta, is a teratogen, and causes fetal anticoagulation throughout the pregnancy. Exposure during early pregnancy can result in embryopathy, while exposure later in pregnancy can cause fetal bleeding, including intracranial hemorrhage. Unfractionated heparin (C) is generally only recommended in the final weeks of the pregnancy because it can be rapidly reversed for delivery. Since it is administered intravenously (unlike low molecular weight heparin which is administered subcutaneously), it is not feasible to maintain an individual on this therapy throughout a pregnancy.

What is the most likely underlying chronic medical problem in the patient with the following ECG? Cardiomyopathy COPD Hyperthyroidism Mitral stenosis

Correct Answer ( B ) Explanation: This electrocardiogram demonstrates multifocal atrial tachycardia, a form of atrial tachycardia diagnosed on the electrocardiogram by three distinct p-wave morphologies. In approximately 60% of cases, patients have underlying pulmonary disease, most commonly COPD. Patients may have a primary cardiac pathology although this is much less common. Cardiomyopathy (A) is not commonly associated with atrial dysrhythmias, but may be the causative agent of ventricular dysrhythmias, particularly ventricular tachycardia. Hyperthyroidism (C) is on the differential diagnosis of atrial fibrillation and should be excluded in cases of new onset atrial fibrillation. Valvular disease, particularly mitral stenosis (D) may also lead to atrial fibrillation as the atrium dilates over time as a result of the stenosis.

A 28-year-old woman who has been breastfeeding her baby for approximately 3 months visits her physician complaining of breast tenderness and fevers. After assessing the patient, the physician believes that she is experiencing mastitis. The patient is started on analgesics and antibiotics. What should the physician recommend in terms of breastfeeding from the affected breast? Continue breastfeeding since the small bacterial load can help develop the infant's immune system Continue breastfeeding to decrease the chance of the mastitis progressing to a breast abscess Discontinue breastfeeding to allow time for proper healing Discontinue breastfeeding to decrease bacterial spread to the infant

Correct Answer ( B ) Explanation: This mother is experiencing mastitis and should be counseled to continue breastfeeding from the affected breast, which decreases the progression of mastitis to breast abscess. Nursing-associated mastitis typically occurs when bacterial organisms from the infant's nasopharynx are transmitted to the mother through a fissure around her nipple or areola. Treatment for mastitis typically includes analgesics, antibiotics, and continued breastfeeding. Staphylococcus aureus is the most common isolated organism. The American Family Physician reports that mastitis most commonly affects breastfeeding women and presents with swelling, pain, and redness in a wedge-shaped area on the breast. While there is no definitive way to prevent it, AFP recommends not wearing tight-fitting bras, attempting to empty all of the areas of the breast while breastfeeding, and airing one's nipples when possible. Treatment includes continued breastfeeding in order to prevent the spread of bacteria to the milk that is left in the breast, pain control, and antibiotics. Discontinuing breastfeeding is not recommended (C) and (D) because breast milk helps improve the infant's immunity and provided the right amount of nutritional support. Breastfeeding also enhances the mother-infant bonding experience. The small bacteria load in the nipple will not help to develop the infant's immune system (A). The infant received passive immunity from direct immunoglobulins in the breast milk.

A 23-year-old man presents with leg pain for 3 months and requests hydromorphone for pain. The nurse approaches you because she believes the patient is "drug seeking" as he became extremely upset when she told him that he would have to wait for the doctor to evaluate him before pain medications could be given. Upon entering the room, the patient speaks pleasantly with you and compliments you on your kindness. He tells you that he only comes to this hospital because "it's the best in the world," and "none of the other doctors understand me." Upon informing the patient that you will not be prescribing hydromorphone, he becomes extremely upset and starts yelling. What personality disorder traits is this patient exhibiting? Antisocial Borderline Histrionic Narcissistic

Correct Answer ( B ) Explanation: This patient exhibits a number of features typically seen in borderline personality disorder. Borderline personality disorder is characterized by unstable relationships, self image and affect. This instability is often marked with impulsiveness. In the Emergency Department, these patients will often "split" providers: they will act with affection and respect to some providers and anger and disregard to others. Substance abuse and drug seeking behavior are often seen in borderline patients as well. Antisocial personality disorder (A) is typified by a disregard and violation of the rights of others. Histrionic personality disorder (C) involves excessive emotionality and attention seeking. Narcissistic personality disorder (D) involves patterns of grandiosity and lack of empathy to others.

A 32-year-old previously healthy man presents to the ED with a 4-hour history of palpitations. He denies chest pain, shortness of breath, or history of similar palpitations. He does admit to heavy alcohol use in the past week, drinking 1 pint of vodka and a 24-pack of beer each day. In the ED, his vital signs are BP 135/75, HR 115, RR 14, and oxygen saturation 98% on room air. An irregularly irregular rhythm is heard on auscultation and an ECG shows atrial fibrillation. What is the next step in management? Chemical cardioversion Observation Rate control Synchronized cardioversion

Correct Answer ( B ) Explanation: This patient has holiday heart syndrome, which can produce atrial fibrillation, atrial flutter, or atrial tachycardia after excessive alcohol use. Patients generally present with palpitations. The rhythm tends to spontaneously convert back to a sinus rhythm within 24-48 hours; thus, the best step in management at this time is to observe the patient with cardiac monitoring. If the patient stays tachycardic beyond 24-48 hours, rate control (C) can be employed, using medications such as calcium channel blockers or beta-blockers. Chemical cardioversion (A) and synchronized cardioversion (D) are both unnecessary now, as the patient is likely to revert to a sinus rhythm on his own. However, if symptoms persist, cardioversion may be used to reduce the risk of thrombus formation in addition to the need for either anticoagulation or a transesophageal echocardiogram. In addition, if the patient becomes unstable at any time, synchronized cardioversion would be used.

A 67-year-old woman presents with severe unilateral pain in her face and head. She characterizes the symptoms as lasting only a few seconds and denies any associated nausea, vomiting, dizziness, or blurry vision. Her Head, Eye, Ear, Nose and Throat Exam (HEENT) and neurologic exam are normal. Which of the following statements is correct regarding this patient's condition? Facial droop is associated with this condition Pain is most commonly located in the V2 and V3 cranial nerve distribution Pain is usually bilateral in the elderly population Poor dentition is associated with this condition The typical pain attack lasts 30-60 minutes then resolves spontaneously

Correct Answer ( B ) Explanation: Trigeminal neuralgia manifests with unilateral facial pain characterized as lancinating paroxysms of pain in the lips, teeth, gums, and chin. The pain is often associated with triggers such as chewing, brushing the teeth, shaving, washing or touching the affected area of the face, swallowing, or exposure to heat or cold in the affected area. Pain most often occurs in the maxillary (V2) and mandibular (V3) distribution of the trigeminal (V) nerve. The ophthalmic division (V1) is rarely involved. Facial droop (A) never accompanies trigeminal neuralgia. Other than acute stroke, conditions associated with facial droop are Bell's palsy (painless) and Ramsey-Hunt syndrome (painful). The pain (C) of trigeminal neuralgia is usually unilateral, a feature independent of age. Poor dentition (D) is not associated with trigeminal neuralgia; however, odontogenic infections may cause unilateral facial pain and should be ruled out in such patients. Pain occurs in clustered episodes (E) that last a few seconds to several minutes. The attacks can occur during the day or night but rarely during sleep.

Which of the following is not a relative contraindication to an arthrocentesis? Anticoagulant usage Bacteremia History of arthrocentesis Infection of the overlying skin Prosthetic joints

Correct Answer ( C ) Explanation: A history of a previous arthrocentesis (C) is not considered a contraindication to joint aspiration. Known bacteremia (B) is considered a relative contraindication as are bleeding diatheses. Anticoagulation (A) use is considered a relative contraindication to arthrocentesis. Infection in the tissues overlying the site to be punctured (D) is generally considered a relative contraindication to arthrocentesis. Prosthetic joints (E) are at high risk for infection and therefore arthrocentesis should be avoided whenever possible. However this is not an absolute contraindication and should be performed if clinically indicated.

A 60-year-old missionary woman is being evaluated for dementia. She also has diarrhea. She was recently treated for tuberculosis with isoniazid. Physical examination shows a symmetric hyperpigmented rash on both arms. Deficiency of which of the following vitamins most likely resulted in this patient's condition? Vitamin B1 Vitamin B12 Vitamin B3 Vitamin B9

Correct Answer ( C ) Explanation: A patient presenting with dementia, dermatitis, and diarrhea, who was recently treated with isoniazid, strongly suggests niacin (vitamin B3) deficiency. The most characteristic finding of niacin deficiency (pellagra) is the presence of a symmetric hyperpigmented rash. Other findings include diarrhea and neurological symptoms (eg, dementia, insomnia, anxiety, delusions, disorientation). Isoniazid (INH) biochemically competes with niacin and inhibits the conversion of tryptophan to niacin, resulting in niacin deficiency. Thiamine (vitamin B1) deficiency (A) typically presents with beriberi (neurological and cardiovascular disease), Wernicke-Korsakoff syndrome (neuro-psychiatric disorder), and optic neuropathy (bilateral visual loss or impaired color perception). Thiamine deficiency is most commonly seen in chronic alcoholics. Folate (vitamin B9) deficiency (C) typically presents with a sore tongue (glossitis), weakness, fatigue, and anemia. Vitamin B12 (cobalamin) deficiency (D) typically presents with paresthesias, ataxia, and megaloblastic anemia (eg, weakness, fatigue, easy bruising).

A 42-year-old man presents to the Emergency Department with nausea, vomiting, and right upper quadrant abdominal pain. He drinks alcohol daily. Which of the following laboratory results would be most consistent with alcoholic hepatitis? Alkaline phosphatase 350 U/L Aspartate transaminase 1000 U/L and alanine transaminase 1200 U/L Aspartate transaminase 250 U/L and alanine transaminase 120 U/L Mean corpuscular volume 60 fL

Correct Answer ( C ) Explanation: Acute hepatitis can be the result of an infectious process (most commonly viral), toxic injury, or alcohol. Alcoholic hepatitis can range from subclinical disease to acute liver failure. Patients present with nausea, vomiting, and RUQ abdominal pain. On examination, they frequently have a tender, enlarged liver, and possibly jaundice. Laboratory studies include a macrocytic anemia and thrombocytopenia. The WBC count is often elevated as is the prothrombin time and bilirubin. Liver transaminases, alanine transaminase (ALT) and aspartate transaminase (AST) are typically elevated 2-10 times normal. Unlike hepatitis due to other causes, AST is predominantly elevated, often with a AST:ALT ratio of 2:1. While patients with alcoholic hepatitis may have elevations of alkaline phosphatase, levels over three to four times normal (alkaline phosphatase 350 U/L) (A) are more typically seen with obstructive etiologies. Serum transaminases over ten times normal (Aspartate transaminase 1000 U/L and alanine transaminase 1200 U/L) (B) are unusual in alcoholic hepatitis and would point to a toxic or viral etiology. A mean corpuscular volume of 60 mL (D) would indicate a microcytic anemia. Chronic alcoholics will have a macrocytic anemia.

A 52-year-old obese man presents to your office with a complaint of right foot pain. He admits that his alcohol consumption has been increasing over the past 5 years and he frequently eats "surf and turf" at his favorite restaurant. He takes daily aspirin and simvastatin. Physical exam reveals redness, warmth, inflammation and exquisite tenderness to palpation of his right great toe. Which of the following provides the most definitive diagnosis? Blood test Clinical diagnosis Synovial fluid analysis Ultrasound

Correct Answer ( C ) Explanation: Gout is characterized by recurrent attacks of inflammatory arthritis due to deposition of monosodium urate crystals. Risk factors for gout include male sex, increasing age, alcohol consumption, obesity, hyperlipidemia, dietary factors, and use of medications that alter urate balance. Synovial fluid analysis in patients with acute gout will show intracellular monosodium urate crystals and provides the most definitive diagnosis of gout. First-line treatment of acute gout involves the use of NSAIDs or colchicine. Prevention includes reduction of risk factors and use of urate-lowering therapy such as allopurinol. Blood test (A) during an acute attack of gout may show nonspecific inflammatory changes and the uric acid level may be low, normal or high. Clinical diagnosis (B) may be made when synovial fluid analysis is not available, however should be considered provisional and is not definitive. Ultrasound (D) can support the diagnosis of gout and is useful in detection of the disease and monitoring of therapy, but does not provide the most definitive diagnosis.

A 16-year-old male presents with a sore throat, adenopathy, and fatigue. He has no evidence of airway compromise. A heterophil antibody test is positive. Appropriate management includes which one of the following? A corticosteroid An antiviral agent Avoidance of contact sports Bed rest

Correct Answer ( C ) Explanation: Infectious mononucleosis presents most commonly with a sore throat, fatigue, myalgias, and lymphadenopathy, and is most prevalent between 10 and 30 years of age. Both an atypical lymphocytosis and a positive heterophil antibody test support the diagnosis, although false-negative heterophile testing is common early in the disease course. The cornerstone of treatment for mononucleosis is supportive, including hydration, NSAIDs, and throat sprays or lozenges. It is also associated with an enlarged spleen. Therefore, patients should be advised to avoid contact- or collision-type activities for 3-4 weeks because of the increased risk of rupture and should avoid physical activity for approximately 6 weeks. The mainstay of treatment for infectious mononucleosis is good supportive care, including adequate hydration, nonsteroidal anti-inflammatory drugs or acetaminophen for fever and myalgias. In general, corticosteroids (A) do not have a significant effect on the clinical course of infectious mononucleosis, and they should not be used routinely unless the patient has evidence of acute airway obstruction. The use of acyclovir and other antiviral agents (B) have shown no consistent or significant benefit, and are not recommended. Bed rest (D) is not recommended; in fact studies have shown that bed rest has slowed recovery and therefore patients are encouraged to return to their usual activities with the exception of contact sports.

Which of the following is the most common type of migraine headache? Basilar-type migraine Migraine with aura Migraine without aura Ophthalmoplegic migraine

Correct Answer ( C ) Explanation: Migraine headaches tend to be familial and affect women twice as often as men. The underlying pathophysiology is thought to be vasogenic inflammation. The first headache usually occurs in an individual in the teens or twenties. The headache is often unilateral, pulsating in quality, moderate to severe in intensity, and exacerbated by routine activities. The side of the headache can vary with individual attacks and may be bilateral up to 40% of the time. The onset is usually gradual and the attacks last from 4-72 hours. The migraine without aura is most common (80%) and is usually with associated nausea, vomiting, photophobia, or phonophobia. Pharmacological treatment is divided into abortive therapies (limit the intensity and duration of an episode) and prophylactic therapies (decrease the frequency and intensity of attacks). Basilar-type migraines (A) are associated with an aura caused by brainstem involvement and are associated with multiple neurologic findings (visual impairment, dysarthria, tinnitus, vertigo, bilateral paresthesias, paresis, and altered mental status). The aura of a classic migraine (B) consists of focal neurologic symptoms that precede and portend the migraine attack. The aura is reversible and lasts 10-20 minutes but may continue up to an hour. The most common aura is visual involving scintillating scotomas (bright rim around an area of visual loss), teichopsias (subjective visual image perceived with eyes open or closed), fortification spectrums (zigzagged wall of fortress slowly drifting across visual field), photopsias (poorly formed brief flashes or sparks of light), and blurred vision. Ophthalmoplegic migraine (D) is a rare syndrome associated with paresis of 1 or more ocular nerves—most commonly CN III (oculomotor). Secondary causes should be evaluated for, including intracranial aneurysm and mass lesion.

Which of the following clinical findings differentiates rheumatoid arthritis from osteoarthritis? Involvement of the proximal interphalangeal (PIP) joints Polyarticular involvement Presence of constitutional symptoms Symmetric joint involvement

Correct Answer ( C ) Explanation: Osteoarthritis (OA) and rheumatoid arthritis (RA) share a number of features but constitutional symptoms are only seen in RA. RA is a chronic inflammatory disease. Patients often present with fever, weakness and musculoskeletal pain lasting for weeks to months. Arthritis involves symmetric joints of the hands, wrists and elbows. Morning stiffness is common and usually last for about an hour. Inflammation may affect other organs leading to hepatitis, scleritis, myocarditis, pericarditis and pleuritis. Atlantoaxial subluxation may also occur and cervical spine precautions should be maintained during intubation. Both RA and OA often involve the PIP joints (A) and can be monoarticular or polyarticular (B). Symmetric joint involvement (D) is common in both RA as well as OA.

A 30-year-old woman with no past medical history presents to the emergency department complaining of substernal chest pain for two hours. It is not worse with exertion and was not relieved by sublingual nitroglycerin. She admits to some mild nausea. She does not smoke cigarettes or use any illicit drugs. Her family history includes a grandmother who died of a myocardial infarction at 84-years-old. Labs in the emergency department are unremarkable. Point of care troponin is negative and ECG reveals sinus rhythm. What is your next step in management? Dobutamine stress test Exercise stress test Reassurance Stress echocardiography

Correct Answer ( C ) Explanation: Reassurance is most appropriate for this patient as her pretest probability for coronary heart disease it is very low. An initial estimate of the likelihood of coronary heart disease is referred to as the pretest probability and is based upon the clinical history. The pretest probability determines the need for noninvasive diagnostic tests or even coronary arteriography. The pretest probability is calculated using patient's age, sex and classification of their angina. Angina is classified as "typical," "atypical" or "nonanginal." Typical angina has three components: (1) sub-sternal pain or discomfort; (2) provoked by exertion or emotional stress; (3) relieved by rest or nitroglycerin or both. Atypical chest pain has two of the three components and nonanginal chest pain has one or none of the three components. In this clinical scenario, the patient is 30 years old, a woman, and has chest pain classified as nonanginal. This gives her a 2% likelihood of having coronary heart disease and stress testing is not indicated. Stress testing is typically needed for initial diagnosis and subsequent clinical management in patient with potential cardiac disease. The utility of cardiac stress testing is determined in part by pretest probability. In patients with low pretest probability, false positive results may lead to potential downstream testing and treatment that may cause more harm than good. Conversely, in patients with high coronary heart disease pretest probability, falsely negative studies lead to false security and missed diagnoses. Therefore, the greatest benefit is in patients with an intermediate pretest probability, in whom a positive test significantly increases the disease likelihood and a negative test significantly decreases the likelihood. Dobutamine stress test (A), exercise stress test (B) and stress echocardiography (D) are different forms of stress testing that have distinct indications including, but not limited to, the patient's ability to exercise, resting electrocardiogram, prior cardiac history, and structural heart status.

A 35-year-old meat cutter comes to your office with persistent symptoms of nausea, vomiting, and diarrhea, which began about 36 hours ago on the last day of a 5-day Caribbean cruise. His wife was sick during the first 2 days of the cruise with similar symptoms. Findings on examination are negative, and a stool specimen is negative for white blood cells. Which one of the following is the most likely cause of his illness? Giardia Hepatitis A Norovirus Rotavirus

Correct Answer ( C ) Explanation: Recent reports of epidemics of gastroenteritis on cruise ships are consistent with Norovirus infections due to waterborne or foodborne spread. In the United States these viruses are responsible for about 90% of all epidemics of nonbacterial gastroenteritis. The noroviruses are common causes of waterborne epidemics of gastroenteritis, and have been shown to be responsible for outbreaks in nursing homes, on cruise ships, at summer camps, and in schools. Symptomatic norovirus gastroenteritis typically develops 24-48 hours after ingestion of contaminated food or water or after contact with an infected individual. The onset can be abrupt or gradual, but each episode is short-lived, lasting only 24-72 hours. The absence of fecal leucocytes and occult blood in stool is helpful in ruling out other enteroinvasive infectious diarrhea processes. Stool culture should be performed to exclude infection with bacterial organisms such as Yersinia, Shigella, Salmonella, and Campylobacter species. Oral fluid and electrolyte replacement is generally adequate for the treatment of norovirus infections. Symptomatic relief can be achieved using antiemetics for nausea and vomiting and analgesics for myalgias and headache. Hepatitis A (B) is a viral illness that is prevalent in developing countries. Patents initially present with flu-like symptoms then later develop jaundice and scleral icterus. Patients at risk of developing acute hepatitis A virus infection should undergo immunization for the virus. Giardia (A) can cause asymptomatic colonization or acute or chronic diarrheal illness. The organism has been found in as many as 80% of raw water supplies from lakes, streams, and ponds. Stool specimen shows Giardia intestinalis trophozoites or cysts via a stool ova and parasite. Rotavirus (D) is a common gastrointestinal virus most commonly seen in children in daycare during the winter months. Following the introduction of the rotavirus vaccine in 2006, a substantial decrease in hospitalization for diarrhea in children younger than 5 years has been observed.

Which of the following statements is true regarding gestational diabetes? Gestational diabetes eventually evolves into Type 2 diabetes Glitazones are the first line treatment Preliminary screening is a blood glucose >130 one hour after a 50 gram glucose load Screening is performed between 32 and 36 weeks

Correct Answer ( C ) Explanation: Screening for gestational diabetes (GDM) is performed between 24 and 28 weeks' gestation with a 50-g oral glucose load and blood glucose measured 1 hour later (abnormal >130 mg/dL). Screening for GDM also identifies women at risk of having a macrosomic baby, the result of excessive glucose crossing the placenta. Those who do not fall into the 'low-risk' pregnant women group are routinely screened, usually before the mid-second trimester, with a 50-g glucose challenge test given under random conditions, and not necessarily fasting. The 'low-risk group' is defined as nonminority women, younger than 25-years-old, with no suggestion of insulin resistance (no prior history of glucose intolerance, obesity, hypertension, or family history of DM). Those patients with a positive 50-g glucose challenge test will then undergo a 100-g, three hour glucose tolerance test, which is diagnostic. Many women who are diagnosed with GDM can be maintained within fasting and postprandial glycemic goals with dietary adjustments and exercise alone. When glycemic goals are not achieved with behavioral therapies, insulin has traditionally been given in increasing dosages. Although high concentrations of insulin overcome the characteristic insulin resistance of the second half of pregnancy, the result is often marked weight gain. Also, human insulin and lispro insulin are the only antidiabetic agents approved for treatment in pregnancy. There are no reports of risk to the fetus from any short- or long-acting synthetic insulin preparations, some clinicians have been reluctant to use long-acting synthetic basal insulin preparations, which mimic basal insulin secretion and limit weight gain, until more outcome data is available. Thus, the role of oral agents in pregnancy has been reconsidered. Complications of GDM include preeclampsia, polyhydramnios, fetal macrosomia with increased risk of birth trauma, neonatal metabolic complications, and increased perinatal morbidity. In about 50% of women, GDM is reversible after pregnancy and does not inevitably evolve into type 2 diabetes (A). Asymptomatic type 1 and type 2 diabetes, or latent autoimmune diabetes of adults existing before pregnancy may become clinical and irreversible. Glitazones (B) might be effective in overcoming the insulin resistance of pregnancy but are contraindicated. Their mechanism of action as transcription factors is a theoretic risk for teratogenesis. Screening for gestational diabetes is performed at 24-28 weeks' gestation not at 32-36 weeks (D)

A 1-year-old boy is brought to the emergency department for severe abdominal pain, vomiting, and diarrhea. His mother says he has had several episodes of intermittent abdominal pain occurring about every 15 to 20 minutes. He appears to be in distress and is continuously crying. Physical examination reveals a "sausage-shaped" abdominal mass palpated on the right side of the abdomen. Which of the following is the most likely diagnosis? Gastroenteritis Hirschsprung disease Intussusception Meckel diverticulum

Correct Answer ( C ) Explanation: The constellation of severe intermittent abdominal pain, vomiting, diarrhea, and a "sausage-shaped" abdominal mass most likely suggests intussusception. Intussusception refers to the invagination (telescoping) of a part of the intestine into itself. It is the most common abdominal emergency in early childhood, particularly in children younger than two years of age. Patients typically present with sudden onset of intermittent, severe, crampy, progressive abdominal pain, accompanied by inconsolable crying. The episodes usually occur at 15 to 20 minute intervals. Palpation of the right side of the abdomen usually reveals a "sausage-shaped" mass. Treatment usually involves surgery for patients that are acutely ill. Gastroenteritis (A) typically presents with diarrhea, vomiting, fever, and abdominal cramps. Hirschsprung disease (B) is a motor disorder of the gut which is caused by the failure of neural crest cells to migrate during intestinal embryogenesis. These patients typically present with symptoms of distal intestinal obstruction (eg, bilious vomiting, abdominal distension, and failure to pass meconium). Meckel diverticulum (D) is considered the most common congenital abnormality of the small bowel. It is caused by incomplete obliteration of the vitelline duct and typically presents with painless rectal bleeding.

You are called to examine a one-week-old boy in the ED for seizures. At home, the mother noticed jittery bilateral leg movements that cannot be stopped when the legs are held. The neonate was born full-term to a 25-year-old G2, P2 mother with limited prenatal care. There were no complications at delivery. At the ED, the vital signs are normal with a normal neurologic examination. You note abnormal facie consisting of a small mouth, cleft palate, low set ears, and a widened distance between the inner canthi with short palpebral fissures. Which of laboratory finding is most consistent with the diagnosis? Anemia Hyperkalemia Hypocalcemia Hypoglycemia

Correct Answer ( C ) Explanation: The infant has an abnormal facie that presents with seizures, which is suspicious for DiGeorge syndrome. This disorder arises from a failure of migration of neural crest cells into the third and fourth pharyngeal pouches. About 80 to 90 percent of patients with DiGeorge syndrome have microdeletions involving chromosome 22q11. Patients typically present in the first week after birth with signs of hypocalcemia such as tetany or seizures that is secondary to hypoplastic or absent parathyroid glands. Characteristic facial features include a small mouth, a submucous cleft palate, abnormal and low set ears, upturned nose, and a widened distance between the inner canthi with short palpebral fissures. Cardiac defects are frequently present and manifest as outflow tract or aortic arch abnormalities such as truncus arteriosus, tetralogy of Fallot, or interrupted aortic arch. The thymic hypoplasia results in an immune defect that is highly variable. Diagnosis is established by fluorescence in situ hybridization with DNA probes specific for 22q11. Anemia, hypoglycemia, and hyperkalemia are not expected findings in DiGeorge syndrome. Common symptoms of anemia (A) include lethargy, tachycardia, and pallor. Symptoms of hyperkalemia (B) include muscle weakness or paralysis and cardiac conduction abnormalities. Hypoglycemia (D) may manifest with jitteriness, hypotonia, and seizures however, hypoglycemia is not found in DiGeorge syndrome.

A 7-year-old girl with no past medical history presents with fever, cough and runny nose. She is well appearing with normal vital signs. Exam reveals clear lungs, nasal congestion and a red, bulging tympanic membrane. What treatment is indicated? Ceftriaxone intramuscular High-dose amoxicillin with clavulinic acid Observation and close follow up without initial antibiotics Polymixin B/Neomycin/Hydrocortisone drops

Correct Answer ( C ) Explanation: This patient presents with an upper respiratory infection and signs of an otitis media (OM). OM is the most common diagnosis made by US physicians in patients <15 years of age but is most common in children <36 months of age. Diagnosis rests on the presence of a middle ear effusion evidenced by absent or limited mobility of the tympanic membrane (TM) or the presence of a bulging TM or an air fluid level behind the TM. Additionally, the middle ear should have inflammation presenting as erythema or pain. It is important to note that the TM can become erythematous if a child is crying. Otitis media may be caused by a number of bacteria but Streptococcus pneumoniae is the most common bacterial pathogen. Other common causative organisms are Haemophilus influenzae and Moraxella catarrhalis. In the past, all OM was treated immediately with antibiotics. However, more recent recommendations recognize that many patients with OM have viral causes and do not require antibiotics. Up to 80% of cases of acute OM resolve spontaneously. The American Academy of Pediatricians currently endorses either antibiotic therapy or observation for select populations (see table below). Severe acute OM is defined as moderate to severe otalgia, otalgia for at least 48 hours or temperature >39oC (102.2°F). If the decision is made to start antibiotics, first line therapy should be with high-dose amoxicillin 80-90 mg/kg/day split into two doses. Ceftriaxone intramuscular (A) and amoxicillin/clavulanic acid (B) are indicated for treatment of acute OM after failure of first-line therapy. Polymyxin B/Neomycin/Hydrocortisone drops (D) are an appropriate treatment for otitis externa (swelling and redness of the external canal) but does not treat otitis media.

Which of the following imaging studies is most appropriate to diagnose toxic megacolon? Colonoscopy Computed tomography Plain radiography Ultrasound

Correct Answer ( C ) Explanation: Toxic megacolon is a complication of infectious colitis or inflammatory bowel disease and is potentially lethal. Patients with toxic megacolon present with signs of systemic toxicity and a nonobstructive colonic dilatation larger than 6 cm. Diagnostic criteria include evidence on plain radiography of colonic dilatation, any three of the following clinical signs: fever, leukocystosis, anemia or tachycardia, and any one of the following: dehydration, hypotension, altered mental status or electrolyte abnormality. The goal of treatment is to restore normal colonic motility and decrease the chance of perforation by reducing the severity of colitis. Initial treatment is medical with complete bowel rest and nasogastric tube placement. Patients should be monitored in the intensive care unit. For patients requiring surgery due to urgent or emergent presentation, subtotal colectomy with end-ileostomy is the procedure of choice. Colonoscopy (A) may be required to determine the level and type of obstruction, but is not used to make the diagnosis of toxic megacolon and is risky due to the possibility of colonic perforation. Computed tomography (B) may help in management, determination of etiology and identifying complications of the disorder, but is not the first line study. Ultrasound (D) may also be used in management, but is not the imaging study of choice for diagnosis

A 17-year-old boy is seen in the clinic because of a skin lesion. He first noted a bump on his penis that became ulcerated. He denies any pain. He admits he is sexually active and has had multiple sexual partners. He denies the use of contraception. On physical exam, there is a 1 centimeter ulcer with a raised, indurated margin on the penile head accompanied by bilateral inguinal lymphadenopathy. Which of the following is the most likely etiologic agent? Haemophilus ducreyi Herpes simplex virus Klebsiella granulomatis Treponema pallidum

Correct Answer ( D ) Explanation: A boy has a single, painless ulcer that is suspicious for primary syphilis. Syphilis is a chronic infection caused by the bacterium Treponema pallidum. Transmission of T. pallidum usually occurs via direct contact with an infectious lesion during sex. It is thought that the spirochete gains access via disrupted epithelium at sites of minor trauma. Primary syphilis occurs after an average incubation period of two to three weeks. A painless papule appears at the site of inoculation that soon ulcerates to produce the classic chancre of primary syphilis, a one to two centimeter ulcer with a raised, indurated margin. The ulcer generally has a non-exudative base and is associated with mild to moderate regional lymphadenopathy that is often bilateral. Such lesions usually occur on the genitalia, but occasionally patients may develop chancres at other sites of inoculation. Chancres heal spontaneously within three to six weeks even in the absence of treatment. Since the ulcer is painless, many patients do not seek medical attention, a feature that enhances the likelihood of transmission. The mechanism of healing is unknown but is thought to be a consequence of local immune responses. While the chancre represents initial local infection with T. pallidum, widespread dissemination of the spirochete also occurs early during the primary stage of infection. The chancre of primary syphilis is best diagnosed by dark field microscopy. Haemophilus ducreyi (A) causes a chancroid that forms a deep, undermined, purulent ulcer that may be associated with painful inguinal lymphadenitis. Herpes simplex virus (B) causes genital herpes characterized by multiple, shallow, and tender ulcers that may be vesicular. Klebsiella granulomatis (C) causes granuloma inguinale (donovanosis), which is a genital infection that is usually sexually acquired and is most prevalent in tropical regions. It causes pseudo-buboes in the inguinal area, which are typically associated with ulcers on the genitalia. Pseudo-buboes are caused by subcutaneous granulation and are eventually broken down and replaced by ulcers.

What is the etiologic agent that causes erysipelas? Haemophilus influenzae Pseudomonas aeruginosa Staphylococcus aureus Streptococcus pyogenes

Correct Answer ( D ) Explanation: Erysipelas is a bacterial skin infection involving the upper dermis that characteristically extends into the superficial cutaneous lymphatics. Lymphatic involvement often is manifested by overlying skin streaking and regional lymphadenopathy. Erysipelas presents as a tender, intensely erythematous, indurated plaque with a sharply demarcated border. This is in opposition to the slightly deeper involvement seen in cellulitis, in which lesions present with limited edema and less well-defined borders. Streptococci pyogenes, also known as group A streptococci, is the primary cause of erysipelas. Most facial infections are attributed to group A streptococci, while an increasing percentage of lower extremity infections are being caused by non-group A streptococci. Streptococcal toxins are thought to contribute to the brisk inflammation that is typical of this infection. Most cases do not have an inciting wound or skin lesion and are preceded by influenza-like symptoms. The incidence of erysipelas is rising, especially in young children, the elderly, persons with diabetes, alcoholic persons, and patients with compromised immune systems or lymphedema. Antibiotics should be started as soon as possible in patients with erysipelas. As previously stated, streptococci cause most cases of the disease; thus, penicillin has remained a first-line therapy. No conclusive evidence demonstrates a pathogenic role for Staphylococcus aureus (C) in typical erysipelas. Staphylococcus aureus is the causative organism for cellulitis, follculitis, impetigo and other serious skin infections such as staphylococus scalded skin syndrome. "Hot tub" folliculitis is caused by Pseudomonas aeruginosa (B) contamination of under-treated water in a hot tub or whirlpool. Multiple pustular or papular perifollicular lesions appear on the trunk and sometimes extremities within six to 72 hours after exposure. Haemophilus influenzae (A) is known to cause cellulitis, specifically buccal and periorbital.

A patient experiences painless bright-red blood per rectum after bowel movements. Perianal examination reveals no abnormalities. You diagnose non-strangulated, large internal hemorrhoids, and then prescribe topical anesthetics and stool softeners. Three weeks later, the patient reports no improvement. At this point in management, which of the following is the most recommended treatment for this patient's non-strangulated internal hemorrhoids? Injection sclerotherapy Lord's procedure Operative hemorrhoidectomy Rubber band ligation

Correct Answer ( D ) Explanation: Hemorrhoids can occur internally and externally. Internal hemorrhoids occur above the dentate line, and as such, do not cause cutaneous pain, but can still prolapse, bleed, become strangulated and necrotic, and cause perianal discomfort. The most common presentation is painless bleeding with bowel movements. Comparatively, external hemorrhoids occur below the dentate line and usually result in acute thrombotic pain, skin tags, bleeding and recurrent hygiene difficulties. Evaluation of any bright-red rectal bleeding usually requires anoscopy or flexible sigmoidoscopy, while colonoscopy is reserved for those with a normal external examination or those with risk factors for colon cancer. Treatment of internal hemorrhoids begins with increased fiber and fluid intake, and avoiding spicy foods and anti-inflammatory medications. Smaller, uncomplicated hemorrhoids may be managed successfully with stool softeners, topical anesthetics, toilet habit retraining, astringents and Sitz baths. Most surgeons use non-operative outpatient therapies, such as rubber band ligation, as treatment. If refractory, or if symptoms are severe, or if the internal hemorrhoid is strangulated, operative hemorrhoidectomy is recommended. Acute thrombosis of an external hemorrhoid usually requires prompt surgical excision, while skin tags and recurrent hygiene difficulties can be managed with outpatient excision procedures. Injection sclerotherapy (A) is not preferred over rubber band ligation in the outpatient, conservative management of hemorrhoids, as it leads to higher rates of post procedure pain, abscess, urinary retention and recurrence. Lord's procedure (B), in which the anal canal is manually stretched under anesthesia, can lead to a disruption of sphincter function. As such, most United States surgeons do not recommend this procedure. Non-strangulated internal hemorrhoids rarely require operative hemorroidectomy (C).

A 6-week-old newborn is being evaluated for projectile vomiting and failure to thrive. The mother tells you the infant vomits immediately after every feeding. The vomitus appears to be nonbilious. Palpation of the right upper quadrant of the abdomen reveals an "olive-like" mass. Laboratory studies show a hypochloremic, metabolic alkalosis. Which of the following is the most likely diagnosis? Duodenal atresia Gastroesophageal reflux Hirschsprung disease Pyloric stenosis

Correct Answer ( D ) Explanation: Nonbilious projectile vomiting in a 6-week-old newborn, is highly suggestive of infantile hypertrophic pyloric stenosis (IHPS). Pyloric stenosis is a narrowing between the distal portion of the stomach (pylorus) and the proximal portion of the duodenum. These newborns classically present with immediate postprandial vomiting and electrolyte imbalances. The hypertrophied pylorus is usually palpated as an "olive-like" mass in the right upper quadrant. This physical examination finding is typically pathognomonic for this condition. Laboratory studies show a hypochloremic, metabolic alkalosis due to the loss of large amounts of gastric acid. In addition to fluid administration and correction of electrolyte imbalances, a pyloromyotomy is usually the definitive form of treatment. Duodenal atresia (A) results from failure of the bowel to recanalize during embryogenesis. These patients typically present with bilious projectile vomiting (as opposed to nonbilious projectile vomiting seen in pyloric stenosis). Gastroesophageal reflux (B) is a normal physiologic process that presents with nonprojectile vomiting and does not result in electrolyte disturbances. Hirschsprung disease (C) is a motor disorder of the gut which is caused by the failure of neural crest cells to migrate during intestinal embryogenesis. These patients typically present with symptoms of distal intestinal obstruction (eg, bilious vomiting, abdominal distension, and failure to pass meconium).

A 39-year-old man presents to the ED complaining of general weakness. He has signs of an upper respiratory infection on exam. His rhythm strip is seen above. Which of the following is the most appropriate next step in management? Administer 325 mg aspirin and send for a troponin Apply transcutaneous pacemakers and admit Consult cardiology Symptomatic care and discharge

Correct Answer ( D ) Explanation: The ECG denotes first degree AV heart block. This is due to prolonged conduction of atrial impulses without the loss of any single impulse. On the ECG, the PR interval will be >200 ms. This type of block is often a normal variant without clinical significance, occurring in close to 2% of healthy young adults and does not require specific treatment. Although the block is benign, in some patients it may be associated with beta-adrenergic blocker therapy, endocarditis, myocarditis, inferior wall myocardial infarction, hyperkalemia, and hypothermia. Treatment for the patient in this case is supportive, and he can be sent home. Aspirin (A) is administered to patients with suspected acute coronary syndrome and is not necessary in this patient. First degree heart block does not progress to more advanced blocks that would require application of a transcutaneous pacemaker (B). Since this block is benign, cardiology (C) does not need to be consulted.

A 55-year-old woman with a history of chronic bronchitis presents to the ED concerned about the appearance of her left eye. She denied a change in vision or any discomfort but is concerned because she noticed "blood in my eye" this morning. Evaluation of the patient demonstrates intact extraoccular muscles; pupils that are equal, round, and reactive to light; and visual acuity of 20/30 in each eye. Which of the following is the most appropriate next step in the evaluation and management of her complaint? CT scan of the orbit and facial bones to evaluate for globe injury Measure intraocular pressure to rule out a retrobulbar hematoma Obtain an ophthalmology consultation Reassure the patient and discharge her home Stain with fluorescein dye and assess for a corneal abrasion

Correct Answer ( D ) Explanation: The patient has a subconjunctival hemorrhage. This usually occurs with trivial events such as coughing or sneezing, Valsalva maneuver, or minor blunt trauma. The hemorrhage appears flat and is limited to the bulbar conjunctiva and stops abruptly at the limbus. This appearance is important to differentiate it from bloody chemosis, which can occur with scleral rupture, coagulopathy, or cavernous sinus thrombosis. Other than its appearance, this condition does not cause the patient any pain or change in visual acuity. It generally heals within 14 days. The patient in this scenario has a history of bronchitis and likely developed the subconjunctival hemorrhage from coughing. No further intervention is necessary and the patient should be reassured and discharged home. CT scan (A) is a valuable tool when there is concern for injury to the orbit or globe. It helps in the evaluation of a foreign body, orbital cellulitis, fracture, globe rupture, or retinal detachment. Intraocular pressure (B) is a valuable measurement when the differential includes acute glaucoma, hyphema, lens dislocation or swelling, suprachoroidal hemorrhage, or retrobulbar hemorrhage. Although it can be considered in individuals at risk for these conditions, it has a limited role in the diagnosis or management of subconjunctival hemorrhage. Indications for ophthalmologic consultation (C) include globe penetration, complex eyelid lacerations, penetrating foreign bodies, injuries to the anterior chamber, hyphema involving > 25% of the anterior chamber, orbital compartment syndrome, or anytime you are concerned or not entirely comfortable with the diagnosis, management, or follow-up plan. Ophthalmology consultation is not necessary for the management of a subconjunctival hemorrhage. Fluorescein staining (E) an eye helps to diagnose a corneal abrasion or ulcer. Patients with corneal injury often complain of light sensitivity with pain and a foreign-body sensation in their eye. This patient may have sustained minor trauma causing her subconjunctival hemorrhage. But absent discomfort, there is limited reason to suspect corneal injury.

A patient presents with chest pain and the ECG seen above. Which of the following medications is contraindicated in this patient's management? Aspirin Clopidogrel Heparin Nitroglycerin

Correct Answer ( D ) Explanation: This patient presents with an inferior ST elevation myocardial infarction (STEMI) and the use of nitroglycerin is relatively contraindicated in management. In patients with myocardial ischemia or infarction, nitrates are used to decreased myocardial oxygen demand. They increase venous capacitance leading to decreased preload and are direct coronary artery vasodilators. Coronary artery vasodilation leads to increased blood flow to ischemic myocardium. The beneficial effects of nitrates are profound leading to their recommendation for most patients with a systolic blood pressure > 90 mm Hg. An inferior STEMI is one of these contraindications. Patients with an inferior STEMI may also have right ventricular infarct and be preload dependent. In a patient with an inferior STEMI, right ventricular infarct is suggested by the presence of ST elevation in lead III larger than that in lead II. A right ventricular infarct can be discovered by performing a right-sided ECG and looking for ST elevation in lead "RV4." In these patients, a preload reducing medication like nitroglycerin can lead to a precipitous drop in blood pressure. Aspirin (A) is the most beneficial treatment in patients with ACS and should be given to all patients unless they have a severe allergy. Clopidogrel (B) is an antiplatelet agent that is beneficial in patients that will be going to cardiac catheterization. Heparin (C) has also been found to be beneficial in STEMI and should only be withheld in patients with concomitant bleeding or a history of hypersensitivity reaction.

A 43-year-old man presents with pain, swelling, and redness to his left leg for 2 days. He denies fever or history of similar presentations in the past. He was hospitalized a month ago for 3 days. Vital signs are unremarkable. Physical examination reveals a 3 cm area of erythema, warmth, and purulence on the left shin. What treatment is recommended? Amoxicillin-Clavulanate Cephalexin Ciprofloxacin Trimethoprim-Sulfamethoxazole (TMP-SMX)

Correct Answer ( D ) Explanation: This patient presents with signs and symptoms of cellulitis requiring antibiotic treatment covering the most likely pathogens. Cellulitis is a soft tissue infection involving the skin and subcutaneous tissue. It is characterized by erythema, swelling, local warmth and tenderness. Cellulitis can occur anywhere on the body but usually presents on the lower extremities, upper extremities and face. Staphylococcus aureus and Streptococcus pyogenes are the most commonly isolated organisms and antibiotics should be directed against these pathogens. The diagnosis is made clinically and there is no specific test that aides in diagnosis. Standard treatment is with immobilization, elevation, warm compresses and antibiotics. In the last 10-15 years, community acquired methicillin resistant S. aureus (CA-MRSA) has become a common causative organism. Cephalexin is a first generation cephalosporin with activity against many streptococcus and staphylococcus species but not against MRSA. TMP-SMX is active against most strains of MRSA but does not have adequate activity against S. pyogenes. Per IDSA guidelines, for outpatients with purulent cellulitis (e.g., cellulitis associated with purulent drainage or exudate in the absence of a drainable abscess), empirical therapy (e.g. TMP-SMX) for CA-MRSA is recommended pending culture results. Empirical therapy for infection due to β-hemolytic streptococci is likely to be unnecessary. Amoxicillin-clavulanate (A) ​does not provide coverage against MRSA and is not recommended in treatment of purulent cellulitis. Cephalexin alone (B) is adequate for non-purulent cellulitis but is inadequate when there is purulence because it is not active against MRSA. Ciprofloxacin (C) is not active against streptococcus species or MRSA.

A 28-year-old woman presents to your office complaining of three days of fever, odynophagia, trismus, and a gradual change in her voice. On examination of her oral cavity, you see an edematous, exudative tonsil with contralateral uvular deviation. She denies any other medical history. Which of the following is the most appropriate initial management strategy for this patient? 24-hour trial of antibiotics only Emergent tonsillectomy Intravenous dexamethasone Needle aspiration and antimicrobial therapy

Correct Answer ( D ) Explanation: This patient's history and physical is consistent with a diagnosis of peritonsillar abscess (PTA), which is a collection of microbial pustular fluid within the tonsil. Most often, peritonsillar abscesses affect children and adolescents and generally presents with the above findings. Since this is essentially a clinical diagnosis, most cases should be treated with drainage (needle aspiration or incision and drainage), hydration, and appropriate antibiotic therapy (gram-positive and respiratory anaerobic organism coverage). While most patients are placed on antibiotics for peritonsillar abscess, an incision and drainage or aspiration is the mainstay of treatment and is more effective than antibiotics alone. Peritonsillar cellulitis can be treated with antibiotics alone (A). Trismus is indicative of an abscess. As such, the patient in the above clinical scenario should undergo drainage. Tonsillectomies (B) are not recommended for routine peritonsillar abscesses unless there is another compelling indication such as airway compromise, previous or recurrent abscess, an uncooperative patient, or failure of initial management. Corticosteroids (C) may reduce the inflammation associated with peritonsillar abscess. However, it should not be administered without antibiotics and drainage.

A 63-year-old woman with a history of a recent viral upper respiratory infection presents with vertigo that started this morning after she woke up. Throughout the day, she experienced intense spinning associated with head movements. She reports nausea and nonbloody, nonbilious emesis. She denies recent trauma and pain. On exam, she has horizontal nystagmus beating towards the left that is exacerbated by changing head positions and resolves after a few moments. She notes decreased hearing on the right. She is otherwise neurologically intact and has an unremarkable head and neck exam. What is the most likely diagnosis? Benign paroxysmal peripheral vertigo Cerebrovascular accident of the posterior circulation Vertebral artery dissection Vestibular labyrinthitis

Correct Answer ( D ) Explanation: Vertigo is a challenging chief complaint that can stem from the central nervous system or the peripheral nervous system. Vertigo is a perception of movement when there is none. Central vertigo is most often due to a cerebrovascular accident of the posterior cerebral circulation. Peripheral vertigo is most often due to cranial nerve VIII or vestibular lesions. When hearing is affected in conjunction with vertigo, it is most likely a peripheral lesion. Vestibular labyrinthitis is inflammation along the vestibulocochlear nerve (CN VIII). It most commonly occurs after a viral illness (e.g. viral URI) or otitis media, lasts for several days, and then often resolves. It is the second most common cause of peripheral vertigo behind benign paroxysmal peripheral vertigo (BPPV). BPPV does not result in hearing loss. Treatment is symptomatic until the condition resolves. Benign paroxysmal peripheral vertigo (A) is a disorder of the semicircular canals in which debris is free floating within the canals. As it only affects the semicircular canals, there is no associated hearing deficit. Cerebrovascular accident of the posterior circulation (B) is a devastating cause of vertigo. There are generally other associated neurologic deficits. Without trauma or neck pain, a vertebral artery dissection (C) is unlikely.


Ensembles d'études connexes

Pelvic Pain and Ectopic Pregnancy

View Set

Education and Health Savings Plans

View Set

Chapter 15 Care of the newborn and infant

View Set

2. Land Use Controls and Regulations

View Set

Solving Systems of Linear Equations: Substitution: Quiz

View Set

Algebra II 3.10: Factor Over the Complex Numbers

View Set